Вы находитесь на странице: 1из 83

American Board of Family Medicine

2012 IN-TRAINING EXAMINATION


CRITIQUE BOOK

This book contains the answers to each question in the In-Training Examination, as well as a critique that
provides a rationale for the correct answer. Bibliographic references are included at the end of each
critique to facilitate any further study you may wish to do in a particular area.

Copyright 2012 The American Board of Family Medicine, Inc. All rights reserved.

Item 1
ANSWER:

Keratoacanthoma is a relatively common lesion in the elderly, but is difficult to distinguish from squamous
cell carcinoma. However, it is easily distinguished from Bowens disease, basal cell carcinoma, Kaposis
sarcoma, and seborrheic keratosis. Most keratoacanthomas undergo a benign self-healing course but may
leave a large, unsightly scar. Treatment is almost always preferred, both for cosmetic reasons and to
prevent the rare case of malignant transformation. Proper treatment for a lesion with this appearance is
excisional biopsy in order to distinguish between keratoacanthoma and squamous cell carcinoma.
Ref:

Habif TP: Clinical Dermatology: A Color Guide to Diagnosis and Therapy, ed 5. Mosby Elsevier, 2010, pp 790-791.

Item 2
ANSWER:

Repeated doses of a short-acting $2-agonist and correction of hypoxia are the main elements of initial
emergency department treatment for acute asthma exacerbations in children. Nebulizer treatments are no
better than a metered-dose inhaler with a spacer (SOR A). High-dose nebulized albuterol every 20 minutes
for 1 hour has not been shown to be beneficial. In children already receiving standard treatment with
albuterol and corticosteroids the addition of intravenous magnesium sulfate has been shown to improve
lung function and reduce the need for hospitalization (SOR A). Oral administration of corticosteroids is
as effective as the intravenous route for reducing the need for hospital admission (SOR A). Mucolytics and
chest physiotherapy have not been shown to be effective in children with acute asthma attacks.
Ref:

Pollart SM, Compton RM, Elward KS: Management of acute asthma exacerbations. Am Fam Physician 2011;84(1):40-47.

Item 3
ANSWER:

Metformin has been found to reduce cardiovascular risk in patients with type 2 diabetes mellitus. It also
decreases the risk of weight gain, and unlike some oral agents it does not significantly increase the risk of
hypoglycemia. It should be continued when insulin is initiated in patients with no renal impairment (SOR
B).
Ref:

Unger J: Diagnosis and management of type 2 diabetes and prediabetes. Prim Care 2007;34(4):731-759. 2) Petznick A:
Insulin management of type 2 diabetes mellitus. Am Fam Physician 2011;84(2):183-190.

Item 4
ANSWER:

Urinary retention is a common problem in hospitalized patients, especially following certain types of
surgery. Starting an "-blocker at the time of insertion of the urethral catheter has been shown to increase
the success of a voiding trial (SOR A). Voiding trial success rates have not been shown to be improved
by leaving the catheter in for 2 weeks, immediate removal of the catheter, using a specialized catheter, or
antibiotic prophylaxis.
Ref:

Selius BA, Subedi R: Urinary retention in adults: Diagnosis and initial management. Am Fam Physician
2008;77(5):643-650.

Item 5
ANSWER:

A (Note: An article on this subject appeared in AFP after the exam was sent to the
printer, and came to our attention after the exam was scored. B is now considered a
better answer, and the critique and references have been updated to reflect this.)

This patients symptoms are consistent with irritable bowel syndrome (IBS). Her history, physical
examination, and laboratory evaluation did not show any evidence of peptic ulcer disease, celiac disease,
thyroid disease, or inflammatory bowel disease. Red flags include unintentional and unexplained weight
loss, rectal bleeding, a family history of bowel or ovarian cancer, and a change in bowel habits to looser
and/or more frequent stools persisting for more than 6 weeks in a person over 60. The patient does not
have any of these findings and therefore does not require any additional testing to confirm the diagnosis
of IBS.
Patients should be given information that explains the importance of self-help in effectively managing their
IBS. This should include information on general lifestyle, physical activity, and dietary intake. A Cochrane
review showed that soluble fiber such as psyllium is not effective for IBS. Lubiprostone is effective for
constipation-predominant IBS.
Ref:

Wilkins T, Pepitone C, Alex B, Schade RR: Diagnosis and management of IBS in adults. Am Fam Physician
2012;86(5):419-426. 2) Ruepert L, Quartero AO, de Wit NJ, et al: Bulking agents, antispasmodic and antidepressants for
the treatment of irritable bowel syndrome. Cochrane Database Syst Rev 2011;(8):CD003460.

Item 6
ANSWER:

Several prospective trials have shown significant benefits from the use of sports-specific training of the hips
and legs in preventing anterior cruciate ligament (ACL) injuries. These programs focus on plyometrics
(repetitive actions that rapidly load and contract a targeted muscle group), strength training, and balance
exercises, along with consistent feedback about proper landing technique. One such study was able to
reduce the frequency of ACL injuries in female high-school soccer players by 88%. Although the risk of
ACL injuries in female athletes is up to 10 times that of males, there is no data to show that restricting their
participation in male-dominated sports is a successful strategy to prevent injuries.
Ref:

Cimino F, Volk BS, Setter D: Anterior cruciate ligament injury: Diagnosis, management, and prevention. Am Fam
Physician 2010;82(8):917-922.

Item 7
ANSWER:

The symptoms of this patient fit the criteria for bronchiectasis, and the gold standard for diagnosis is
high-resolution chest CT. The chest film does not suggest pulmonary tuberculosis, so a PPD would not
be appropriate initially (although tuberculosis can be a cause of bronchiectasis). Bronchoscopy may
eventually be necessary, but not at this point in the investigation. This patients age, the absence of findings
of emphysema, and the lack of a family history of emphysema or liver disease make the diagnosis of
"1-antitrypsin deficiency unlikely.
Ref:

Barker AF: Bronchiectasis. N Engl J Med 2002;346(18):1383-1393. 2) Goldman L, Schafer AI (eds): Goldmans Cecil
Medicine, ed 24. Elsevier Saunders, 2011, pp 548-549.

Item 8
ANSWER:

While there are several guidelines for the clinical diagnosis of acute bacterial sinusitis (ABS), there is
general agreement that patients with a duration of symptoms of at least 10 days without improvement
should be treated with antibiotics, including both children and adults (SOR C). Signs and symptoms may
include nasal drainage and congestion, facial pressure and/or pain, sinus tenderness, and headache.
Recommendations for the duration of treatment vary.
One set of guidelines calls for empiric treatment with amoxicillin alone; another recommends going directly
to amoxicillin/clavulanate. Suggested alternatives include a respiratory quinolone or the combination of
a third-generation cephalosporin and clindamycin, particularly in patients with penicillin allergy. Due to
the increasing emergence of resistant Streptococcus and Haemophilus species, neither trimethoprim/sulfamethoxazole nor macrolides are now recommended for empiric treatment of ABS.
Data regarding the efficacy of other measures such as nasal irrigation and the use of decongestants is
limited and variable. The most recent guidelines do not recommend the use of decongestants, whether oral
or topical.
Ref:

Aring AM, Chan MM: Acute rhinosinusitis in adults. Am Fam Physician 2011;83(9):1057-1063. 2) Chow AW, Benninger
MS, Brook I, et al; Infectious Diseases Society of America: IDSA clinical practice guideline for acute bacterial
rhinosinusitis in children and adults. Clin Infect Dis 2012;54(8):e72-e112.

Item 9
ANSWER:

This patient has classic lichen planus, with pruritic, symmetrically distributed papular lesions. The
violaceous flat-topped papules, usually 36 mm in size, are distinct and so characteristic in appearance that
a biopsy is usually not necessary to make the diagnosis. First-line treatment is with high-potency topical
corticosteroids such as clobetasol, as mid-potency topical agents such as triamcinolone are ineffective.
Topical calcineurin inhibitors, including tacrolimus, can be used in cases not responding to topical
corticosteroids. While scabies can masquerade as a variety of other dermatoses, retreatment with a
scabicide is not indicated in this patient.
Ref:

Usatine RP, Tinitigan M: Diagnosis and treatment of lichen planus. Am Fam Physician 2011;84(1):53-60.

Item 10
ANSWER:

Because speech-language therapy is effective for primary expressive language disorders, referral as early
as possible is critical (SOR A). Red flags suggesting the need for immediate evaluation include no babbling
in a 12-month-old, not saying mama or dada at 18 months, a vocabulary of less than 25 words at age
2, and using less than 200 words at age 3. Children should be able to follow two-step commands by 2
years of age (SOR A).
Ref:

McLaughlin MR: Speech and language delay in children. Am Fam Physician 2011;83(10):1183-1188.

Item 11
ANSWER:

Pulmonary arteriovenous malformations are found in 15%30% of patients with hereditary hemorrhagic
telangiectasia (HHT), also known as Osler-Weber-Rendu syndrome. All patients with possible or
confirmed HHT should be screened for pulmonary arteriovenous malformations with contrast
echocardiography (SOR C). While contrast echocardiography is used to detect atrioseptal and ventricular
septal defects, neither of these conditions is particularly prevalent in HHT. Aortic aneurysms and
myocardial perfusion defects are also not associated with HHT.
Ref:

Olitsky SE: Hereditary hemorrhagic telangiectasia: Diagnosis and management. Am Fam Physician 2010;82(7):785-790.

Item 12
ANSWER:

When a patient presents with sudden hearing loss it is important to distinguish between sensorineural and
conductive hearing loss. Patients should be asked about previous episodes, and the workup should include
both an assessment for bilateral hearing loss and a neurologic examination. Sudden sensorineural hearing
loss is diagnosed by audiometry demonstrating a 30-decibel hearing loss at three consecutive frequencies,
with no other cause indicated from the physical examination.
Evaluation for retrocochlear pathology may include auditory brainstem response, MRI, or follow-up
audiometry. Routinely prescribing antiviral agents, thrombolytics, vasodilators, vasoactive substances, or
antioxidants is not recommended. Oral corticosteroids may be offered as initial therapy, and hyperbaric
oxygen therapy may be helpful within 3 months of diagnosis. The guidelines also strongly recommend
against routine laboratory tests or CT of the head as part of the initial evaluation.
Ref:

Stachler RJ, Chandrasekhar SS, Archer SM, et al: Clinical practice guideline: Sudden hearing loss. Otolaryngol Head Neck
Surg 2012;146(3 Suppl):S1-S35.

Item 13
ANSWER:

Hemoglobin A1c (HbA1c) levels of 6.0%6.5% indicate an increased risk for diabetes mellitus, and levels
>6.5% can be used to diagnose diabetes. Hemoglobinopathies and conditions causing hemolysis can cause
HbA1c measurements to be falsely low. The opposite effect is seen in African-Americans, who have higher
HbA1c levels than whites along the continuum of glycemia. Other tests should be used in patients with
conditions that affect HbA1c, including pregnancy.
Ref:

Buysschaert M, Bergman M: Definition of prediabetes. Med Clin North Am 2011;95(2):289-297.

Item 14
ANSWER:

Vocal cord dysfunction is a disorder in which the vocal cords move toward midline during inspiration or
expiration, leading to varying degrees of obstruction. It is commonly misdiagnosed as exercise-induced
asthma. There are a number of precipitating factors, including exercise, psychological conditions, irritants,
rhinosinusitis, and gastroesophageal reflux disease. Spirometry generally will show a normal expiratory
loop with a flattened inspiratory loop. In asthma and COPD the FEV1/FVC ratio is decreased, resulting
in a concave shape in the expiratory portion of the flow-volume curve. The inspiratory loops are generally
normal. Patients with restrictive lung disease have a normal FEV1/FVC ratio with a reduced FVC.
Ref:

Deckert J, Deckert L: Vocal cord dysfunction. Am Fam Physician 2010;81(2):156-159. 2) Mason RJ, Broaddus VC,
Martin TR, et al: Murray & Nadels Textbook of Respiratory Medicine, ed 5. Saunders Elsevier, 2010, pp 1059-1062.

Item 15
ANSWER:

Most cases of uncomplicated acute pyelonephritis, including the one described here, can be managed in
the outpatient setting. Findings that might prompt consideration of inpatient management include comorbid
conditions (e.g., renal dysfunction, urologic disorders, diabetes mellitus, advanced liver or cardiac
disease), hemodynamic instability, male sex, metabolic derangements, pregnancy, severe pain, a toxic
appearance, an inability to take liquids by mouth, or a temperature >39.4C (103.0F).
Nitrofurantoin for 5 days is an appropriate treatment for an uncomplicated urinary tract infection, but not
for pyelonephritis. Amoxicillin is generally not considered first-line treatment for pyelonephritis because
of a high prevalence of resistance to oral $-lactam antibiotics, and it should only be chosen if susceptibility
results for the urinary isolate are known and indicate likely activity. Fluoroquinolones, such as
ciprofloxacin, are the preferred empiric antibiotic treatment for outpatient treatment of pyelonephritis, as
long as the local prevalence of resistance to community-acquired Escherichia coli is 10%.
Ref:

Colgan R, Williams M, Johnson JR: Diagnosis and treatment of acute pyelonephritis in women. Am Fam Physician
2011;84(5):519-526.

Item 16
ANSWER:

The attending physician is responsible for determining capacity and incapacity for decision making. The
extent, cause, and probable duration of any incapacity should be documented in the clinical record.
Ref:

Olick RS: Defining features of advance directives in law and clinical practice. Chest 2012;141(1):232-238.

Item 17
ANSWER:

Certain drugs can affect taste more than smell, but this does not include the bisphosphonates. Olfactory
disorders may be associated with deficiencies of vitamins A, B6, B12, and trace metals, but not with vitamin
D deficiency. Celiac disease is not known to cause a decreased ability to smell. Rare tumors involving the
olfactory region of the brain can affect smell, and are best detected by MRI.
Ref:

Bromley SM: Smell and taste disorders: A primary care approach. Am Fam Physician 2000;61(2):427-436. 2) Daniels C,
Gottwald B, Pause BM, et al: Olfactory event-related potentials in patients with brain tumors. Clin Neurophysiol
2001;112(8):1523-1530. 3) Mainland JD, Johnson BN, Khan R, et al: Olfactory impairments in patients with unilateral
cerebellar lesions are selective to inputs from the contralesional nostril. J Neurosci 2005;25(27):6362-6371.

Item 18
ANSWER:

The American Heart Association recommends a goal blood pressure of 130/80 mm Hg or less for the
treatment of hypertension in patients with diabetes mellitus, chronic kidney disease, or coronary artery
disease.
Ref:

Pflieger M, Winslow BT, Mills K, Dauber IM: Medical management of stable coronary artery disease. Am Fam Physician
2011;83(7):819-826.

Item 19
ANSWER:

Effective treatments for chronic orthostatic hypotension include fludrocortisone, midodrine, and
physostigmine (SOR B). Clonidine, pseudoephedrine, terbutaline, and theophylline are not appropriate
therapies.
Ref:

Lanier JB, Mote MB, Clay EC: Evaluation and management of orthostatic hypotension. Am Fam Physician
2011;84(5):527-536.

Item 20
ANSWER:

Dysthymic disorder is characterized by depressed mood for at least 2 years in addition to at least two of
the following: change in appetite, alteration in sleep, low energy, low self-esteem, poor concentration,
or feelings of hopelessness. There must be no history of a manic or hypomanic episode, substance abuse,
a chronic psychotic disorder, or an organic cause.
Symptoms of major depression are similar to those of dysthymia and can occasionally be difficult to
distinguish from dysthymia. This patients lifelong history of a depressed mood not triggered by any
particular depressing event, and the predominance of patient complaints as opposed to objective signs,
indicate that major depression is not the diagnosis in this case.

Bipolar disorder is characterized by major depression with periods of mania. Cyclothymia is characterized
by dysthymia with periods of hypomania. Adjustment disorder with depressed mood is characterized by
impaired social or occupational functioning or abnormal symptoms within 3 months of a stressor.
Ref:

Sadock BJ, Sadock VA, Ruiz P (eds): Kaplan & Sadocks Comprehensive Textbook of Psychiatry, ed 9. Lippincott Williams
& Wilkins, 2009, pp 1711-1713.

Item 21
ANSWER:

This patient has the classic pulmonary manifestations of Ascaris infection, which develop during the
transpulmonary passage of Ascaris larvae (SOR C). The larvae produce a syndrome of transient
eosinophilic pulmonary infiltrates, commonly referred to as Lffler syndrome. Ascaris infection is the most
common worldwide cause of this syndrome. Symptoms develop when larvae are within the lungs,
approximately 912 days after ingestion of Ascaris eggs. Patients may develop the following symptoms
and signs:

an irritating, nonproductive cough and burning substernal discomfort


dyspnea and blood-tinged sputum
urticaria during the first few days of the illness (15% of patients)
fever, which infrequently exceeds 38.3C (101.0F)
crackles and wheezing, with no signs of consolidation
hepatomegaly

The acute symptoms generally subside within 510 days, depending upon the severity of the illness. The
chest radiograph may show round or oval infiltrates ranging in size from several millimeters to several
centimeters in both lung fields; these lesions are more likely to be present when blood eosinophilia exceeds
10%. The infiltrates are migratory and may become confluent in perihilar areas, and usually clear
completely after several weeks.
Taenia does not infiltrate the lungs, but forms cysts in the muscles. Diphyllobothrium latum, the fish
tapeworm, does not cause pulmonary problems. Enterobius (pinworm) does not migrate from the
gastrointestinal tract into other organs.
Ref:

Parasites - Ascariasis. Centers for Disease Control and Prevention, 2010. 2) Eckstein B: Primary care for refugees. Am
Fam Physician 2011;83(4):429-436.

Item 22
ANSWER:

Nipple pain with breastfeeding is extremely common, with some studies reporting a prevalence of up to
96%. Preventing or alleviating nipple pain is important for comfort, but also for promoting breastfeeding
in general. The best intervention for nipple pain is education on proper positioning and attachment of the
infant. Topical remedies may also be effective, although no one topical agent has been shown to be clearly
superior, and none is as effective as education on positioning and latch-on.
Ref:

Lochner JE, Livingston CJ, Judkins DZ: Clinical inquiries: Which interventions are best for alleviating nipple pain in
nursing mothers? J Fam Pract 2009;58(11):612a-612c.

Item 23
ANSWER:

Also known as erythema infectiosum or fifth disease, parvovirus B19 infection is a fairly common cause
of an exanthematous rash and arthritis in younger women. This infection should be particularly suspected
in health-care workers who have frequent contact with children. The specific characteristics of the rash,
the pattern of joint involvement, and the place of employment in an otherwise healthy person all offer clues
suggesting parvovirus B19 as the infecting agent. Measles virus, adenovirus, and HIV rarely cause
arthritis, although HIV infection can cause a musculoskeletal syndrome later in the disease.
Varicella-zoster virus may cause large-joint arthritis, but the rash is distinctively vesicular and pruritic.
Ref:

Longo DL, Fauci AS, Kasper DL, et al (eds): Harrisons Principles of Internal Medicine, ed 18. McGraw-Hill, 2012, pp
1478-1480.

Item 24
ANSWER:

Diagnostic paracentesis is recommended for patients with ascites of recent onset, as well as for those with
chronic ascites who present with new clinical findings such as fever or abdominal pain. A neutrophil count
>250/mL is diagnostic for peritonitis. Once peritonitis is diagnosed, antibiotic therapy should be started
immediately without waiting for culture results. Bloody ascites with abnormal cytology may be seen with
hepatoma but is not typical of peritonitis. The ascitic fluid pH does not become abnormal until well after
the neutrophil count has risen, so it is a less reliable finding for treatment purposes. A protein level >1
g/dL is actually evidence against spontaneous bacterial peritonitis.
Ref:

Longo DL, Fauci AS, Kasper DL, et al (eds): Harrisons Principles of Internal Medicine, ed 18. McGraw-Hill, 2012, pp
1077-1078, 2518-2519.

Item 25
ANSWER:

There are a number of alternatives to the bisphosphonates. Unfortunately, efficacy data is not encouraging
for most of them. Intravenous zoledronic acid has been shown to reduce both hip fracture risk and
vertebral fracture risk. Teriparatide reduces vertebral fracture risk but not hip fracture risk. The same is
true for raloxifene and calcitonin salmon.
Ref:

Favus MJ: Bisphosphonates for osteoporosis. N Engl J Med 2010;363(21):2027-2035.

Item 26
ANSWER:

The syndrome of inappropriate secretion of antidiuretic hormone should be suspected in any patient who
has hyponatremia and excretes urine that is hypertonic relative to plasma. A urine sodium concentration
>20 mEq/L combined with a low BUN level provides further support for the diagnosis. Additional
findings may include weakness, lethargy, mental confusion, and weight gain.

Sodium depletion usually causes clinical features of dehydration, an elevated BUN level, and a urine
sodium concentration <20 mEq/L. Primary polydipsia almost invariably results in dilute urine with low
osmolality when compared to serum. Renal failure is unlikely with a BUN level of 4 mg/dL. Adrenal
insufficiency is also unlikely, as most patients will have skin pigmentation, weight loss, and hypotension.
A normal serum potassium level is also inconsistent with the diagnosis of adrenal insufficiency.
Ref:

Goldman L, Schafer AI (eds): Goldmans Cecil Medicine, ed 24. Elsevier Saunders, 2011, pp 730-732. 2) Taal MW,
Chertow GM, Marsden PA, et al: Brenner & Rectors The Kidney, ed 9. Elsevier Saunders, 2011, pp 576-580.

Item 27
ANSWER:

Endotracheal suctioning of vigorous infants born through meconium-stained amniotic fluid is not
recommended (SOR C). Although infants born through thick meconium are more likely to develop
aspiration syndrome, endotracheal suctioning does not provide any benefit over expectant management in
preventing this condition or other respiratory problems. Endotracheal suctioning may be useful if the
infant is not vigorous or shows signs of respiratory depression.
Suctioning of the infants stomach can be done electively but is not required for immediate management.
Positive pressure ventilation is indicated for ventilatory support of newborns with respiratory depression
who are not born through meconium-stained amniotic fluid.
Ref:

Wiswell TE, Cannon CM, Jacob J, et al: Delivery room management of the apparently vigorous meconium-stained neonate:
Results of the multicenter, international collaborative trial. Pediatrics 2000;105(1 Pt 1):1-7. 2) Raghuveer TS, Cox AJ:
Neonatal resuscitation: An update. Am Fam Physician 2011;83(8):911-918.

Item 28
ANSWER:

When family dynamics lead to conflict during an office visit, it is best for the physician to attempt to
remain neutral by avoiding triangulation, which occurs when the two sides in conflict each attempt to align
with a third party. Priority should be given to the patients right to privacy and confidentiality, and the
physician should ask permission from the patient to discuss his or her health issues with other people.
Physicians should always remember who they are primarily responsible to.
Ref:

Omole FS, Sow CM, Fresh E, et al: Interacting with patients family members during the office visit. Am Fam Physician
2011;84(7):780-784.

Item 29
ANSWER:

DRESS is an acronym for Drug Reaction with Eosinophilia and Systemic Symptoms. The hallmark of
DRESS syndrome is erythroderma accompanied by fever, lymphadenopathy, elevation of liver enzymes,
and eosinophilia. The offending medication should be discontinued immediately and treatment with
corticosteroids should be initiated. Seizure medications such as carbamazepine, phenytoin, lamotrigine,
and phenobarbital are responsible for approximately one-third of cases. Allopurinol-associated DRESS
syndrome has the highest mortality rate.

Toxic shock syndrome should be suspected in patients with erythroderma, hypotension, and laboratory
evidence of end-organ involvement (elevated liver enzymes or kidney function studies, anemia,
thrombocytopenia, or elevation of creatine kinase). Treatment with intravenous clindamycin, which inhibits
toxin synthesis, should be undertaken immediately.
Stevens-Johnson syndrome is characterized by a vesiculobullous rash with mucocutaneous involvement,
and erysipelas is a painful localized rash with well-demarcated borders. Red man syndrome is associated
with vancomycin.
Ref:

Cacoub P, Musette P, Descamps V, et al: The DRESS syndrome: A literature review. Am J Med 2011;124(7):588-597.
2) McQueen A, Martin SA, Lio PA: Derm emergencies: Detecting early signs of trouble. J Fam Pract 2012;61(2):71-78.

Item 30
ANSWER:

In cases of drug overdose, several critical physical findings must be evaluated. The most important is the
size of the pupils. Tolerance rarely reduces the miotic effects of narcotic medications. A patient who is
comatose, with decreased breathing, a slow pulse, and small pupils should be strongly suspected of having
overdosed on a narcotic. Naloxone should be administered to reverse these effects. The response to
treatment with naloxone is irregular. Cerebral infarction in the pontine angle, organophosphate poisoning,
phenothiazine overdose, and treatment for glaucoma can also cause constricted pupils, but these
associations are seen much less frequently than narcotics overdose.
Ref:

Longo DL, Fauci AS, Kasper DL, et al (eds): Harrisons Principles of Internal Medicine, ed 18. McGraw-Hill, 2012, p
3554.

Item 31
ANSWER:

Alcoholic ketoacidosis generally occurs in a patient who has been drinking heavily without eating. Blood
glucose levels are usually low or normal, and volume depletion associated with nausea, vomiting, and
abdominal pain is the norm. Patients typically have high osmolal and anion gaps. Treatment of alcoholic
ketoacidosis includes vigorous volume repletion with normal saline, along with administration of thiamine
and glucose. Only in the rare presence of marked acidemia (pH <7.10) is the administration of
bicarbonate thought to be necessary. Though insulin levels may be low, hyperglycemia is seldom found.
N-acetylcysteine and pyridoxine are not used for the treatment of alcoholic ketoacidosis. Levels of
glucagon and hydrocortisone are typically elevated in patients with alcoholic ketoacidosis.
Ref:

Marx JA (ed): Rosens Emergency Medicine: Concepts and Clinical Practice, ed 7. Mosby Elsevier, 2010, pp 2375-2392.
2) Melmed S, Polonsky KS, Larsen PR, Kronenberg HM (eds): Williams Textbook of Endocrinology, ed 12. Elsevier
Saunders, 2011, pp 1436-1461.

10

Item 32
ANSWER:

Magnesium sulfate has a long history of use for preventing seizures in preeclampsia and eclampsia, and
a recent Cochrane review confirmed that it is the preferred agent. Benzodiazepines and fosphenytoin are
secondary agents that can be used if magnesium sulfate fails, but they are not as effective. Nimodipine was
also shown to be less effective than magnesium sulfate. Delivery is indicated, but magnesium sulfate must
also be administered (SOR A).
Ref:

Duley L, Glmezoglu AM, Henderson-Smart DJ, Chou D: Magnesium sulphate and other anticonvulsants for women with
pre-eclampsia. Cochrane Database Syst Rev 2010;(11):CD000025. 2) Fogleman CD: Magnesium sulfate and other
anticonvulsants for women with preeclampsia. Am Fam Physician 2011;83(11):1269-1270.

Item 33
ANSWER:

Medicare eligibility for home oxygen therapy is based on oxygen saturation. To qualify for continuous
long-term oxygen therapy the patient must have a PaO2 55 mm Hg or an SaO2 88 mm Hg.
Ref:

Oxygen therapy supplies: Complying with documentation & coverage requirements. Centers for Medicare & Medicaid
Services, Medicare Learning Network, 2011.

Item 34
ANSWER:

Sialadenosis, bilateral noninflammatory enlargement of the parotid gland, is associated with diabetes
mellitus. Periodontal bleeding and inflammation, candidiasis, and delayed wound healing also are
associated with diabetes mellitus.
Tooth erosion can be an oral manifestation of gastroesophageal reflux disease or bulimia. Cobblestone oral
mucosa is seen in Crohns disease. Diffuse melanin pigmentation is an oral finding of Addisons disease.
Painful oral ulcers occur in several conditions, including Behet syndrome, aphthous ulcers, pemphigus,
and pemphigoid.
Ref:

Chi AC, Neville BW, Krayer JE, Gonsalves WC: Oral manifestations of systemic disease. Am Fam Physician
2010;82(11):1381-1388.

11

Item 35
ANSWER:

This lesion is suspicious for melanoma, based on the asymmetry, irregular border, color variegation, and
size larger than 6 mm. In addition, a history of evolution of the lesion, with changes in size, shape, or
color, has been shown in some studies to be the most specific clinical finding for melanoma. The preferred
method of biopsy for any lesion suspicious for melanoma is complete elliptical excision with a small
margin of normal-appearing skin. The depth of the lesion is crucial to staging and prognosis, so shave
biopsies are inadequate. A punch biopsy of the most suspicious-appearing area is appropriate if the location
or size of the lesion makes full excision inappropriate or impractical, but a single punch biopsy is unlikely
to capture the entire malignant portion in larger lesions. Electrodesiccation and curettage is not an
appropriate treatment for melanoma. Mohs surgery is sometimes used to treat melanomas, but is not used
for the initial diagnosis.
Ref:

Shenenberger DW: Cutaneous malignant melanoma: A primary care perspective. Am Fam Physician 2012;85(2):161-168.

Item 36
ANSWER:

Most episodes of pharyngitis are caused by viral rather than bacterial infections. The use of clinical
decision rules for diagnosing group A $-hemolytic streptococcal pharyngitis improves quality of care while
reducing unwarranted treatment and overall cost (SOR A). The original Centor score used four signs and
symptoms to estimate the probability of acute streptococcal pharyngitis in adults with a sore throat, and
was later modified by adding age as a fifth criterion. One point each is assigned for (1) absence of cough,
(2) swollen, tender anterior cervical nodes, (3) temperature >38.0C (100.4F), and (4) a tonsillar
exudate and swelling. One point is added for patients between the ages of 3 and 14 years, and a point is
subtracted for patients over the age of 45. The cumulative score determines the likelihood of streptococcal
pharyngitis and the need for antibiotics, and guides testing strategies. Patients with a score of zero or 1 are
at very low risk for streptococcal pharyngitis and do not require testing or antibiotic therapy. Patients with
a score of 23 should be tested using a rapid antigen test or throat culture, and a positive result warrants
antibiotic therapy. Patients with a score of 4 or higher are at high risk for streptococcal pharyngitis, and
empiric treatment may be considered. This patients score is zero, and no testing or treatment is warranted.
Ref:

Choby BA: Diagnosis and treatment of streptococcal pharyngitis. Am Fam Physician 2009;79(5):383-390. 2) Wessels MR:
Streptococcal pharyngitis. N Engl J Med 2011;364(7):648-655.

Item 37
ANSWER:

The understanding of fibromyalgia has been rapidly expanding in recent years, primarily due to the use
of functional magnetic resonance imaging (fMRI). The hallmark of fibromyalgia is an exaggerated
response to painful stimuli, or an attribution of pain to a stimulus that is normally not painful. Early
research focused on peripheral tissues as the source of this condition. However, the cause has now been
recognized as an abnormality in the central nervous system, which can be seen on fMRI. This physical
brain abnormality differentiates fibromyalgia from psychogenic conditions such as conversion disorder and
malingering.

12

Any evidence of an actual abnormality of the peripheral tissue on physical examination, blood tests, or an
imaging study in a patient suspected to have fibromyalgia should raise the suspicion that another diagnosis
is also present. It is estimated that up to 25% of patients who have a definable rheumatologic condition
such as lupus or rheumatoid arthritis also suffer from fibromyalgia. Many patients who develop
fibromyalgia started having symptoms in the wake of a viral infection (especially Epstein-Barr virus).
However, these viral illnesses are believed to trigger a genetic predisposition rather than being a necessary
cause of this condition.
Ref:

Clauw DJ: Fibromyalgia: An overview. Am J Med 2009;122(12 Suppl):S3-S13. 2) Clauw DJ, Arnold LM, McCarberg
BH; FibroCollaborative: The science of fibromyalgia. Mayo Clin Proc 2011;86(9):907-911.

Item 38
ANSWER:

Slipped capital femoral epiphysis is often misdiagnosed, as the symptoms are frequently vague. It is the
most common hip disorder in adolescents, with the age range being 9 to 15 years. It occurs when the
proximal femoral epiphysis slips posteriorly and inferiorly on the femoral neck through the growth plate.
The typical presentation is a limping child who may have pain in the groin, hip, thigh, or knee. Very often
the pain is vague and poorly localized. It occurs more often in boys, with African-Americans and Pacific
Islanders having a higher rate of involvement, possibly due to increased levels of obesity in these
population groups.
Physical findings vary, depending on the severity of the slippage. A child with a severe slip may not be
able to bear weight. Obligatory external rotation of the involved hip is noted when the hip is passively
flexed to 90. Radiographs are needed to diagnose unstable slipped capital epiphysis, and should include
frog-leg lateral views and anteroposterior views of both hips.
Another cause of hip pain in adolescent patients is apophyseal avulsion fractures. Clinical features include
pain after a sudden, forceful movement. Hip apophysitis presents as activity-related hip pain with a history
of overuse and negative radiographs. In children under the age of 10 years, transient synovitis is also a
common cause of hip pain. It occurs after a viral illness and is associated with negative radiographs but
positive laboratory tests. Fractures may be seen in children on occasion, but there will be a history of
trauma. Septic arthritis is an infrequent cause of hip pain in children, but patients have a history of fever
with elevation of the WBC count and inflammatory joints. The diagnosis would be confirmed by joint
aspiration. Legg-Calv-Perthes disease is also infrequent, and features include vague hip pain with
decreased internal rotation of the hip. The diagnosis is based on findings from radiographs or MRI.
Ref:

Sawyer JR, Kapoor M: The limping child: A systemic approach to diagnosis. Am Fam Physician 2009;79(3):215-224. 2)
Peck D: Slipped capital femoral epiphysis: Diagnosis and management. Am Fam Physician 2010;82(3):258-262.

13

Item 39
ANSWER:

It can be difficult to determine the point at which changes of normal aging are more appropriately
considered disease processes. Although the direction of expected change is generally well understood,
variables such as the level of fitness and overall health of an individual affect the degree of change. As the
body ages, the measured left ventricular ejection fraction, heart rate variability, and maximum heart rate
trend downward, the walls of the major aorta and major arteries stiffen, and the vasodilator capacity of
most smaller vessels is reduced (SOR A). The arterial wall changes increase peripheral resistance and
result in an increase in blood pressure. Positive adaptive changes have been shown in older adults who
engage in regular aerobic exercise, however, and these changes can be measured after only 3 months of
moderate-intensity exercise (SOR A).
Ref:

American College of Sports Medicine, Chodzko-Zajko WJ, Proctor DN, et al: American College of Sports Medicine
position stand: Exercise and physical activity for older adults. Med Sci Sports Exerc 2009;41(7):1510-1530.

Item 40
ANSWER:

Older patients are at higher risk for hypoglycemia caused by oral antidiabetic agents. Glyburide is
associated with a significantly greater risk of symptomatic hypoglycemia than other second-generation
sulfonylurea hypoglycemic agents.
Metformin decreases liver production of glucose and is not associated with hypoglycemia. Even so, this
patients creatinine elevation is a contraindication to metformin use, as it increases the risk of lactic
acidosis.
Glimepiride, glipizide, and repaglinide stimulate insulin release and increase the risk of hypoglycemia.
However, the risk of symptomatic hypoglycemia is substantially lower compared to the risk associated with
glyburide in patients with similar hemoglobin A1c values (SOR B).
Ref:

Gangji AS, Cukierman T, Gerstein HC, et al: A systematic review and meta-analysis of hypoglycemia and cardiovascular
events: A comparison of glyburide with other secretagogues and with insulin. Diabetes Care 2007;30(2):389-394. 2)
Nathan DM, Buse JB, Davidson MB, et al: Medical management of hyperglycemia in type 2 diabetes: A consensus
algorithm for the initiation and adjustment of therapy: A consensus statement of the American Diabetes Association and
the European Association for the Study of Diabetes. Diabetes Care 2009;32(1):193-203. 3) Rogers M, Sands C, Self TH:
Sulfonylurea-induced hypoglycemia: The case against glyburide. Consultant 2011;51(2):97-99.

Item 41
ANSWER:

Acute low back pain is one of the most common presenting symptoms in family medicine practices. In the
absence of red flags such as fever, a history of cancer, or neurologic deficits, patients can be successfully
treated with conservative therapy. Interventions that have been shown to be beneficial include
non-benzodiazepine muscle relaxers (SOR A). They are most effective in the first 12 weeks but can be
used for up to 4 weeks. Additional beneficial treatments include physical therapy, acetaminophen, and
NSAIDs. Bed rest is inadvisable for patients with low back pain (SOR A). Patients who stay active have
better outcomes than those who stay at rest.

14

There is no good evidence that oral corticosteroids are beneficial for acute back pain, and insufficient
evidence that massage therapy is effective. Lumbar traction provides no benefit in acute low back pain
(SOR B).
Ref:

Casazza BA: Diagnosis and treatment of acute low back pain. Am Fam Physician 2012;85(4):343-350.

Item 42
ANSWER:

Gastroesophageal reflux disease is one of the most common causes of chronic cough. Patients with silent
gastroesophageal reflux may not have the classic symptoms of heartburn and regurgitation. The diagnosis
is based on resolution of the cough with an empiric trial of a proton pump inhibitor, although a chest
radiograph should be obtained in all patients with a chronic cough to exclude bronchiectasis, tuberculosis,
and sarcoidosis. Asthma is another frequent cause of chronic cough, but it can be ruled out with normal
pulmonary function tests.
Ref:

Benich JJ III, Carek PJ: Evaluation of the patient with chronic cough. Am Fam Physician 2011;84(8):887-892.

Item 43
ANSWER:

The American College of Cardiology Foundation/American Heart Association guidelines for early
cardiovascular assessment do not recommend lipoprotein and apolipoprotein levels. A C-reactive protein
level can help to determine the need for statin therapy in men 50 and older and women 60 and older whose
LDL-cholesterol levels are <130 mg/dL and who are not on lipid-lowering medication, hormone therapy,
or immunosuppressive therapy, and who do not have clinical coronary heart disease, diabetes mellitus,
chronic kidney disease, severe inflammatory disease, or contraindications to statins. A C-reactive protein
level may also be reasonable in younger patients with intermediate, but not low, cardiovascular risk.
Measurement of cardiac calcium levels is reasonable in patients whose cardiovascular risk is intermediate
(10-year risk 10%20%) or low-to-intermediate (10-year risk 6%10%). An ankle-brachial index is
reasonable for intermediate-risk, but not low-risk, patients. At this point in time, the patient described here
does not meet any recommended criteria for further testing.
Ref:

Randel A: ACCF/AHA release guidelines for early cardiovascular risk assessment. Am Fam Physician 2011;84(2):234-235.
2) Greenland P, Alpert JS, Beller GA, et al: 2010 ACCF/AHA guideline for assessment of cardiovascular risk in
asymptomatic adults: Executive summary: A report of the American College of Cardiology Foundation/American Heart
Association Task Force on Practice Guidelines. Circulation 2010;122(25):2748-2764.

Item 44
ANSWER:

The threshold for transfusion of red blood cells should be a hemoglobin level of 7 g/dL in adults and most
children.
Ref:

Sharma S, Sharma P, Tyler LN: Transfusion of blood and blood products: Indications and complications. Am Fam Physician
2011;83(6):719-724.

15

Item 45
ANSWER:

This patient demonstrates classic findings for acute respiratory distress syndrome (ARDS). In many cases
ARDS must be differentiated from heart failure. Heart failure is characterized by fluid overload (edema),
jugular venous distention, a third heart sound, an elevated BNP level, and a salutary response to diuretics.
A BNP level <100 pg/mL can help rule out heart failure (SOR A). In addition, a patient with ARDS
would not have signs of left atrial hypertension and overt volume overload.
Hypersensitivity pneumonitis is usually preceded by exposure to an inciting organic antigen such as bird
feathers, mold, or dust. Pulmonary embolus, while certainly in the differential, is unlikely to cause such
dramatic radiographic findings. Pneumothorax would be seen on the chest radiograph.
Ref:

Cullan A, Grover M, Hitchcock K: FPINs clinical inquiries: Brain natriuretic peptide for ruling out heart failure. Am Fam
Physician 2011;83(11):1333-1334. 2) Saguil A, Fargo M: Acute respiratory distress syndrome: Diagnosis and management.
Am Fam Physician 2012;85(4):352-358.

Item 46
ANSWER:

Aggressive neurosurgical intervention is not indicated to evacuate clots in patients with intracerebral
hemorrhage except in those with a cerebellar hemorrhage, which is always an indication for neurosurgical
consultation. Guidelines have been developed by the American Heart Association for lowering blood
pressure in patients with a systolic blood pressure >180 mm Hg, or a mean arterial pressure >130 mm
Hg. The use of various forms of osmotherapy, including mannitol, to prevent the development of cerebral
edema has not been shown to improve outcomes. The data regarding hypothermia induction is unclear.
Patients with an INR >1.5 should receive therapy to replace vitamin Kdependent factors and have their
warfarin withheld.
Ref:

Elliott J, Smith M: The acute management of intracerebral hemorrhage: A clinical review. Anesth Analg
2010;110(5):1419-1427. 2) Nyquist P: Management of acute intracranial and intraventricular hemorrhage. Crit Care Med
2010;38(3):946-953.

Item 47
ANSWER:

Blood flow to the kidney is autoregulated so as to sustain pressure within the glomerulus. This is influenced
by angiotensin IIrelated vasoconstriction. ACE inhibitors can impair the kidneys autoregulatory function,
resulting in a decreased glomerular filtration rate and possibly acute renal injury. This is usually reversible
if it is recognized and the offending agent stopped. NSAIDs can exert a similar effect, but they can also
cause glomerulonephritis and interstitial nephritis. Statins, haloperidol, and drugs of abuse (cocaine,
heroin) can cause rhabdomyolysis with the release of myoglobin, which causes acute renal injury.
Thrombotic microangiopathy is a rare mechanism of injury to the kidney, and may be caused by
clopidogrel, quinine, or certain chemotherapeutic agents.
Ref:

Naughton CA: Drug-induced nephrotoxicity. Am Fam Physician 2008;78(6):743-750.

16

Item 48
ANSWER:

Medical therapy with progestational drugs is the treatment of choice for menorrhagia due to endometrial
hyperplasia without atypia. Progestins convert the proliferative endometrium to a secretory one, causing
withdrawal bleeding and the regression of hyperplasia. The most commonly used form is cyclic oral
medroxyprogesterone, given 14 days per month, but implanted intrauterine levonorgestrel is the most
effective (SOR A) and also provides contraception.
High-dose estrogen supplementation would further stimulate the endometrium. Estrogen is useful in cases
where minimal estrogen stimulation is associated with breakthrough bleeding. The anti-fibrinolytic agent
tranexamic acid prevents the activation of plasminogen and is given at the beginning of the cycle to
decrease bleeding. Side effects and cost limit this treatment option, however. It may be most useful in
women with bleeding disorders or with contraindications to hormonal therapy.
NSAIDs, which decrease prostaglandin levels, reduce menstrual bleeding but not as effectively as
progestins. While mefenamic acid is marketed for menstrual cramps and bleeding, all NSAIDs have a
similar effect in this regard.
If medical management fails, hysteroscopic endometrial ablation is an option for reducing uterine bleeding
but is considered permanent and obviously will impair fertility. Hysterectomy is reserved for severe and
chronic bleeding that is not relieved by other measures.
Ref:

Sweet MG, Schmidt-Dalton TA, Weiss PM, Madsen KP: Evaluation and management of abnormal uterine bleeding in
premenopausal women. Am Fam Physician 2012;85(1):35-43.

Item 49
ANSWER:

Osgood-Schlatter disease is an inflammatory condition that is a common cause of knee pain in children and
adolescents. The diagnosis is usually based on clinical findings, although radiographs may be necessary
to rule out fractures or other problems if findings are not typical. MRI, ultrasonography, and orthopedic
referral are not usually needed. The problem is typically self-limited and responds to activity modification,
over-the-counter analgesics, stretching, and physical therapy.
Ref:

Atanda A Jr, Shah SA, OBrien K: Osteochondrosis: Common causes of pain in growing bones. Am Fam Physician
2011;83(3):285-291.

Item 50
ANSWER:

The result of neonatal varicella infection can be catastrophic, with a fatality rate approaching 30%.
Maternal immunity is ideal, but since varicella vaccination is contraindicated during pregnancy the best
alternative is advising the patient to avoid contact with infected individuals until safe postpartum
immunization is possible. Maternal varicella infection is particularly problematic during weeks 1320 of
pregnancy (resulting in a 2% risk of congenital varicella in the newborn) and when the onset of maternal
symptoms occurs from 5 days before until 2 days after delivery.

17

Administration of varicella immune globulin to the expectant mother has not been shown to benefit the
fetus or infant, but because pregnancy can increase the risk of serious complications in the mother the
Advisory Committee on Immunization Practices (ACIP) recommends that administration to pregnant
women be considered following known exposure. The ACIP also recommends that term infants born within
the 7-day window described above, as well as all preterm infants, receive varicella immune globulin, and
that those who develop any signs of varicella infection also be given intravenous acyclovir. Term infants
delivered more than 5 days after the onset of maternal varicella are thought to have adequate passive
immunity for protection and the expected benign course generally requires only observation.
Ref:

Duff P: Diagnosis and management of varicella infection in pregnancy. Perinatology 2010;1(1):6-12. 2) Atkinson W,
Hambersky J, Stanton A, Wolfe C (eds): Epidemiology and Prevention of Vaccine-Preventable Diseases, ed 12. Centers
for Disease Control and Prevention, 2011, pp 301-324.

Item 51
ANSWER:

Fluoroquinolones such as ciprofloxacin have been shown to significantly reduce the duration and severity
of travelers diarrhea when given for 13 days. Sulfacetamide is available only in a topical form for use
in the eye. Penicillin and erythromycin are not effective against the most common cause of travelers
diarrhea, enterotoxigenic Escherichia coli.
Ref:

Mandell GL, Bennett JE, Dolin R (eds): Mandell, Douglas, and Bennetts Principles and Practice of Infectious Diseases,
ed 7. Churchill Livingstone, 2009, pp 1365-1367, 4025-4026.

Item 52
ANSWER:

The primary indication for joint replacement surgery in patients with osteoarthritis is intractable pain,
which is almost always relieved by the surgery. Joint replacement may also be appropriate for patients
with significant limitations of joint function or with altered limb alignment. Range of motion, joint laxity,
and recurrent subluxation relate to musculotendinous function, and are not reliably improved by joint
replacement.
Ref:

Goldman L, Schafer AI (eds): Goldmans Cecil Medicine, ed 24. Elsevier Saunders, 2011, p 1675. 2) Longo DL, Fauci
AS, Kasper DL, et al (eds): Harrisons Principles of Internal Medicine, ed 18. McGraw-Hill, 2012, p 2836.

Item 53
ANSWER:

Heart murmurs are common in children and adolescents. Often the murmur is innocent, but it may also
be the only finding in an asymptomatic child with structural heart disease. Physical findings that should
lead one to consider evaluation for structural heart disease include increased intensity with standing, a
holosystolic murmur, a grade of 3 or higher, a harsh quality, an abnormal S2, maximal intensity at the
upper left sternal border, a diastolic murmur, or a systolic click.

18

Characteristics that are more likely to be associated with innocent murmurs include a systolic murmur, a
soft sound, a short duration, a musical or low pitch, intensity that varies with phases of respiration,
increased loudness in the supine position, and increased loudness with exercise, anxiety, or fear. If the
diagnosis of an innocent murmur cannot be made from physical findings, an echocardiogram is the most
appropriate study. A chest radiograph and EKG rarely assist in the diagnosis of heart murmurs in children
(SOR B) and should not routinely be ordered.
Ref:

Frank JE, Jacobe KM: Evaluation and management of heart murmurs in children. Am Fam Physician 2011;84(7):793-800.

Item 54
ANSWER:

Dog and cat bite wounds may appear trivial, but if they are not managed appropriately they can become
infected and may result in functional impairment. Cultures are recommended for wounds that are clinically
infected. Because it can be toxic to tissue, povidone-iodine surgical scrub should not be used. Irrigation
with either normal saline or Ringers lactate solution may reduce the rate of infection by up to twentyfold.
Tetanus immune globulin is not needed, and DTaP is not given to children 7 years of age or older.
Ref:

Tintinalli JE, Kelen GD, Stapczynski JS (eds): Emergency Medicine: A Comprehensive Study Guide, ed 7. McGraw-Hill,
2011, pp 353-355.

Item 55
ANSWER:

Intravenous antibiotics, especially imipenem, have been shown to be beneficial in patients with pancreatitis.
Patients with pancreatitis who are not vomiting do not require nasogastric tube placement. Corticosteroids
are not indicated in the management of acute pancreatitis, and pseudocysts can be managed initially with
percutaneous aspiration.
Ref:

Bassi C, Larvin M, Villatoro E: Antibiotic therapy for prophylaxis against infection of pancreatic necrosis in acute
pancreatitis. Cochrane Database Syst Rev 2003;(4):CD002941. 2) Al-Omran M, Albalawi ZH, Tashkandi MF, Al-Ansary
LA: Enteral versus parenteral nutrition for acute pancreatitis. Cochrane Database Syst Rev 2010;(1):CD002837.

Item 56
ANSWER:

Conservative treatment of grade 1 and 2 ankle sprains in athletes, consisting of the use of leg casts for 2
weeks followed by progressive increases in activity, has been found to lead to a loss of playing time of 46
weeks. Treatment consisting of an elastic wrap and use of a crutch until pain resolves produces similar
results. Early mobilization after aggressive control of inflammation is recommended. Typical treatment
includes extensive icing, compression, and elevation, followed by the application of air or gel splints. In
the first 48 hours, physical therapy begins with early mobilization, strengthening, and proprioception
retraining. In one study utilizing this more aggressive approach, athletes were able to return to functional
status in 9 days after grade 1 sprains and in 12 days after grade 2 injuries.
Ref:

Marx JA (ed): Rosens Emergency Medicine: Concepts and Clinical Practice, ed 7. Mosby Elsevier, 2010, pp 677-679.
2) Bope ET, Kellerman RD (eds): Conns Current Therapy 2012. Elsevier Saunders, 2012, pp 595-596.

19

Item 57
ANSWER:

Cognitive problems are often a feature of depression in older patients, which can make it difficult to
distinguish depression from dementia. Congruence of mood with delusions is more typical of depression.
A longer duration of cognitive problems, slow progression of cognitive problems, and no past history of
psychiatric problems are more typical of dementia.
Ref:

Michels TC, Tiu AY, Graver CJ: Neuropsychological evaluation in primary care. Am Fam Physician 2010;82(5):495-502.

Item 58
ANSWER:

Sitagliptin is a DPP-4 inhibitor. These agents slow the inactivation of incretin hormones, prolonging their
action and thereby increasing insulin release and decreasing glucagon. Sitagliptin decreases hemoglobin
A1c levels by 0.7%, but there is no data on patient-oriented outcomes or long-term safety with this
medication.
Ref:

Richter B, Bandeira-Echtler E, Bergerhoff K, Lerch CL: Dipeptidyl peptidase-4 (DPP-4) inhibitors for type 2 diabetes
mellitus. Cochrane Database Syst Rev 2008;(2):CD006739. 2) Kripke C: Dipeptidyl-peptidase-4 inhibitors for treatment
of type 2 diabetes. Am Fam Physician 2009;79(5):372.

Item 59
ANSWER:

Pleural fluid is associated with a dull-to-flat percussion note, decreased-to-absent tactile fremitus, and
decreased-to-absent breath sounds. A consolidation would be indicated by bronchial breath sounds and
increased fremitus. Emphysematous blebs and pneumothorax are hyperresonant to percussion.
Ref:

Goldman L, Schafer AI (eds): Goldmans Cecil Medicine, ed 24. Elsevier Saunders, 2011, pp 607-611.

Item 60
ANSWER:

Intranasal corticosteroids are the most effective treatment for mild to moderate allergic rhinitis and should
be first-line therapy. Second-line therapies that can be used for symptoms that do not respond to initial
treatment include antihistamines, decongestants, cromolyn, and leukotriene receptor antagonists.
Nonpharmacologic measures that may be helpful include nasal irrigation and avoiding irritants.
Ref:

Sur DK, Scandale S: Treatment of allergic rhinitis. Am Fam Physician 2010;81(12):1440-1446.

20

Item 61
ANSWER:

Drugs such as lithium, thiazide diuretics, sex hormones, and vitamins A and D can increase the serum
ionized calcium level. The gastrointestinal symptoms associated with lithium toxicity are also the most
common presenting symptoms of hypercalcemia. Hypercalcemic patients may also complain of
constipation, fatigue, lethargy, polyuria, and weakness, all the result of an increased serum level of ionized
calcium (roughly calculated to be 40% of the total serum calcium level plus 0.8 g/dL for each 1 g/dL
decrease in serum albumin below 4 g/dL). The most common causes of hypercalcemia are malignancy and
hyperparathyroidism, together accounting for over 80% of all cases. Excessive ingestion of antacids can
result in milk-alkali syndrome, another cause of hypercalcemia. Granulomatous disease and renal diseases
are other possible causes.
Although additional details are required in this case to determine the cause, primary hyperparathyroidism
augmented by medications is highly likely given the patients age and sex, and measurement of her
parathyroid hormone level must be included in the workup. No matter the cause, the treatment of
symptomatic hypercalcemia should be immediate and directed at lowering the serum calcium level. The
safest and most effective way to accomplish this is with intravenous normal saline volume replacement,
reducing the need for reabsorption of salt, water, and, coincidentally, calcium in the proximal tubules.
Because hypercalcemia often results in volume depletion, aggressive fluid replacement is often ideal,
provided there is no contraindication to doing so. Once the volume depletion is corrected the addition of
loop diuretics such as furosemide can facilitate excretion of calcium. Each of the other options has a place
in the longer term treatment of hypercalcemia in appropriate situations: bisphosphonates for malignancy,
glucocorticoids for granulomatous disease, and cinacalcet for hyperparathyroidism.
Ref:

Taniegra ED: Hyperparathyroidism. Am Fam Physician 2004;69(2):333-339. 2) Moe SM: Disorders involving calcium,
phosphorus, and magnesium. Prim Care 2008;35(2):215-237.

Item 62
ANSWER:

Metformin is well tolerated and there is good data to show it helps prevent type 2 diabetes mellitus in
high-risk patients. Pioglitazone has been shown to slow the progression from prediabetes to diabetes, but
it has more side effects and is more expensive than metformin. Neither glipizide nor exenatide is currently
recommended as a treatment for prediabetes. Acarbose has a high discontinuation rate due to side effects.
Ref:

Ratner RE, Sathasivam A: Treatment recommendations for prediabetes. Med Clin North Am 2011;95(2):385-395. 2)
American Diabetes Association: Standards of medical care in diabetes2012. Diabetes Care 2012;35(Suppl 1):S11-S63.

Item 63
ANSWER:

While spondylolysis occurs in 6% of the general population, it may be the cause of 50% of back pain in
young adults. This unilateral or bilateral vertebral defect of the pars interarticularis is likely due to
repetitive hyperextension of the posterior spine that results in a fracture or stress injury. This usually
occurs at L4-L5. Sports that put increased demands on the spine include football, gymnastics,
weightlifting, soccer, volleyball, and ballet.

21

The recommended initial study for athletes with back pain of more than 3 weeks duration is lumbar spine
radiographs, including anterior/posterior, lateral, and oblique views bilaterally. The Scotty dog with a
collar sign can be noted on the oblique view. This may not be present in early spondylolysis, so a SPECT
scan may be appropriate.
Treatment for spondylolysis consists of discontinuing the offending activity, medication for pain, physical
therapy, and possibly bracing. Healing may take 912 months.
Ref:

Cassas KJ, Cassettari-Wayhs A: Childhood and adolescent sports-related overuse injuries. Am Fam Physician
2006;73(6):1014-1022.

Item 64
ANSWER:

This childs presentation is highly suspicious for pertussis, given the severe coughing paroxysms and the
possibility of inadequate immunization. Two weeks of oral erythromycin is recommended for children with
mild to moderate illness, principally to halt the spread of the infection. Ribavirin is used for respiratory
syncytial virus infection, which is generally seen in much younger children with more respiratory distress.
The cough of pertussis often lasts several weeks. Immune globulin is not recommended.
Ref:

Kliegman RM, Stanton BF, Geme JW III, et al (eds): Nelson Textbook of Pediatrics, ed 19. Elsevier Saunders, 2011, pp
944-948.

Item 65
ANSWER:

Crystalloids are the essential component of fluid resuscitation in patients with severe burn injuries, with
lactated Ringers solution being the most commonly used. Substantial early loss of blood is unusual, and
transfusions are not often required. The use of colloids in these patients has not been shown to be helpful
and may be harmful. Hypertonic saline solution may be useful in selected patients but requires careful
monitoring and may be detrimental.
Ref:

Brunicardi FC, Anderson D, Billiar T, et al (eds): Schwartzs Principles of Surgery, ed 9. McGraw-Hill, 2009, pp 200-201.

Item 66
ANSWER:

Patients with hoarseness lasting longer than 2 weeks with risk factors for dysplasia or carcinoma, such as
smoking, heavy alcohol use, or long-standing gastroesophageal reflux disease, should be evaluated with
laryngoscopy. Inhaled corticosteroids can contribute to hoarseness.
Ref:

Feierabend RH, Shahram MN: Hoarseness in adults. Am Fam Physician 2009;80(4):363-370.

22

Item 67
ANSWER:

Otitis media is a major health problem in the United States; it is the number one reason children visit
doctors and accounts for one-fourth of all antibiotic prescriptions. With appropriate antibiotics most
patients will improve in 23 days. Persistence or worsening of symptoms requires immediate reevaluation,
since complications such as bacterial resistance or meningitis may be developing. Occasionally a persistent
middle ear effusion will be found on reexamination 1014 days after initial treatment. Inflation of the
eustachian tube using the method of Politzer or employing the Valsalva maneuver has been shown to be
ineffective, as have antihistamines and systemic steroids. Most asymptomatic effusions with mild hearing
loss will clear in 90 days if left alone.
Ref:

Kliegman RM, Stanton BF, Geme JW III, et al (eds): Nelson Textbook of Pediatrics, ed 19. Elsevier Saunders, 2011, pp
2207-2209.

Item 68
ANSWER:

The diagnosis of occupational asthma can be made when both bronchospasm and its relationship to the
work environment can be demonstrated. A history of cough, wheezing, chest tightness, or episodic
dyspnea in varying combinations or singly should lead one to suspect bronchospasm. Relating
bronchospasm to the work environment can be done in several ways. A history of exposure to a known
sensitizer is helpful, as is a pattern of symptoms occurring after exposure. With many agents the onset of
symptoms may be delayed up to several hours. A 10% decrease in FEV1 measured before and after a work
shift supports the diagnosis. Improvement of bronchospasm with removal from exposure also suggests the
diagnosis. Treatment includes both standard pharmacologic therapy and removal from exposure as soon
as possible.
Hypersensitivity pneumonitis is an immune-mediated syndrome that is not as common as occupational
asthma. It begins with malaise, fever, and myalgias 46 hours after exposure to an antigen to which the
person has become sensitized. Byssinosis is due to exposure to the dust of hemp, flax, or cotton.
Symptoms vary from reversible chest tightness on one or more days early in the work week to chronic
bronchitis and permanent obstructive lung disease. Toxic pneumonitis or pulmonary edema is the result
of very high exposure to irritant gases, metal dust, or metal fumes, usually associated with unusual
circumstances such as a fire, explosion, or spill. Benign pleural effusions are the most common sequela
during the first 20 years after asbestos exposure. The diagnosis is one of exclusion, made by ruling out
other causes of exudative effusions in workers with known asbestos exposure.
Ref:

Goldman L, Schafer AI (eds): Goldmans Cecil Medicine, ed 24. Elsevier Saunders, 2011, pp 567-569.

23

Item 69
ANSWER:

When self-management and standard management of anticoagulation therapy are compared,


self-management improves the rate of minor hemorrhage, with no difference in the rate of major
hemorrhage. Self-monitoring also improves the rate of thromboembolism. Both self-monitoring and
self-management improve the rate of all-cause mortality. When studied, patients who self-managed their
anticoagulation therapy perceived greater self-efficacy compared to patients receiving standard care, and
self-management did not increase their levels of anxiety. When all factors are considered, self-monitoring
and self-management have outcomes superior to those of standard monitoring and management.
Ref:

Cayley WE Jr: Self-monitoring and self-management of anticoagulation therapy. Am Fam Physician 2011;84(3):266-268.

Item 70
ANSWER:

The serum 25-hydroxyvitamin D level is the best indicator of overall vitamin D status because it reflects
total vitamin D from dietary intake and sunlight, as well as conversion from adipose stores in the liver.
Measurement of 1,25-dihydroxyvitamin D, the active form of vitamin D formed in the kidney, may be
necessary in advanced chronic kidney disease. 24,25-Dihydroxyvitamin D is not biologically active.
Phosphate and parathyroid hormone are involved in the regulation of vitamin D levels, but are not helpful
in determining overall vitamin D status.
Ref:

Rosen CJ: Vitamin D insufficiency. N Engl J Med 2011;364(3):248-254.

Item 71
ANSWER:

Currently, nonalcoholic fatty liver disease is the leading cause of transaminase elevations, and is becoming
increasingly common as obesity becomes more prevalent. It is estimated that some 30% of adults in the
United States have this disease. Patients with metabolic syndrome, diabetes mellitus, or elevated
triglycerides are at the highest risk. If the AST/ALT ratio is >2, especially if (-glutamyl transpeptidase
is elevated, alcoholic liver disease should be suspected.
It is well known that severe hepatotoxicity can occur with acetaminophen overdoses, and dosages of even
4 g/day for 510 days will cause enzyme elevations in more than half of healthy nondrinkers. Herbal
preparations associated with elevated liver enzymes include kava and germander. Hepatitis C can cause
transient enzyme elevations, typically of ALT.
If liver enzymes remain elevated on a repeat test 24 weeks later, the patient should be tested for hepatitis
B and C, and iron, iron binding capacity, and ferritin levels should be ordered to check for
hemochromatosis. A lipid profile and glucose level should be ordered as well, and abdominal
ultrasonography considered to look for evidence of fatty infiltration of the liver.
Ref:

Oh RC, Hustead TR: Causes and evaluation of mildly elevated liver transaminase levels. Am Fam Physician
2011;84(9):1003-1008.

24

Item 72
ANSWER:

Of the common herbal supplements, St. Johns wort interacts with the most drugs, including statins,
warfarin, and antidepressants. The other herbal supplements listed do not interact with statins. Ginkgo
biloba and ginseng may interact with warfarin.
Ref:

Gardiner P, Phillips R, Shaughnessy AF: Herbal and dietary supplementDrug interactions in patients with chronic
illnesses. Am Fam Physician 2008;77(1):73-78.

Item 73
ANSWER:

An elevated level of parathyroid hormone (or a level that is in an unexpected normal range) in a patient
with an elevated calcium level generally indicates a diagnosis of primary hyperparathyroidism. However,
these laboratory findings may also occur with lithium or thiazide use, tertiary hyperparathyroidism
associated with end-stage renal failure, or familial hypocalciuric hypercalcemia, and a medical and family
history should be obtained to assess these possibilities. The other medications listed do not cause
hypercalcemia.
Ref:

Marcocci C, Cetani F: Primary hyperparathyroidism. N Engl J Med 2011;365(25):2389-2397.

Item 74
ANSWER:

According to the 2011 update of the American Heart Association/American College of Cardiology
Foundation guidelines on secondary prevention of coronary artery disease, metoprolol succinate has the
best evidence for mortality reduction when compared to the other medications listed. The other medications
have utility, but in more specialized circumstances: losartan for those intolerant of ACE inhibitors,
clopidogrel for those intolerant of aspirin, and ezetimibe for those intolerant of statins. Spironolactone has
evidence of benefit post myocardial infarction when added to a regimen that includes an ACE inhibitor and
a $-blocker.
Ref:

Smith SC Jr, Benjamin EJ, Bonow RO, et al: AHA/ACCF secondary prevention and risk reduction therapy for patients with
coronary and other atherosclerotic vascular disease: 2011 update: A guideline from the American Heart Association and
American College of Cardiology Foundation. Circulation 2011;124(22):24582473.

Item 75
ANSWER:

The FDA states that antipsychotics are not indicated for treating dementia-related psychosis. The reason
for this is that the efficacy for antipsychotics has not been consistently shown in clinical trials and, in fact,
patients treated with olanzapine functioned worse after treatment than did those who received a placebo.
There is also evidence that these drugs may increase mortality from infection or heart-related conditions.
Practice guidelines recommend the use of antipsychotics only after other options have been exhausted and
symptoms are severe, persistent, and not responsive to nonpharmacologic interventions (SOR B).
Ref:

DEmpaire I, Macalaso M: Psychoses. FP Essentials monograph series, no 385, 2011, pp 27-31.

25

Item 76
ANSWER:

It is thought that easier access to prescription medications leads to a higher incidence of misuse by
physicians. The drugs most commonly abused are benzodiazepines and opioids. Most studies suggest that
alcoholism rates among physicians approximate those of the general population when adjusted for
socioeconomic status. The lifetime prevalence of depression also is similar for physicians and the general
population (12.8% for men and 19.5% for women). Physicians are less likely to abuse illicit drugs,
probably because of their access to prescription drugs.
Ref:

OConnor PG, Spickard A Jr: Physician impairment by substance abuse. Med Clin North Am 1997;81(4):1037-1052. 2)
Center C, Davis M, Detre T, et al: Confronting depression and suicide in physicians: A consensus statement. JAMA
2003;289(23):3161-3166. 3) Baldisseri MR: Impaired healthcare professional. Crit Care Med 2007;35(2 Suppl):S106-S116.

Item 77
ANSWER:

Campylobacter enterocolitis in children is generally a mild, self-limiting disease. However, in patients who
are sick enough to require hospitalization or who remain symptomatic by the time a bacteriologic diagnosis
has been made, antibiotic therapy is indicated. The preferred drug is oral erythromycin, which clinical
trials indicate may produce clinical improvement. Ciprofloxacin may be an effective alternative to
erythromycin in the treatment of Campylobacter, but it is contraindicated in young children. There is no
evidence that ampicillin, trimethoprim, or metronidazole is effective for this disease.
Ref:

Mandell GL, Bennett JE, Dolin R (eds): Mandell, Douglas, and Bennetts Principles and Practice of Infectious Diseases,
ed 7. Churchill Livingstone, 2009, pp 2798-2799.

Item 78
ANSWER:

Protamine sulfate is the treatment of choice for heparin overdose or significant bleeding secondary to
heparin therapy. Vitamin K is used for reversal of anticoagulation from warfarin. Vasopressin is a pressor
agent used to treat hypotensive episodes. Dabigatran is an anticoagulant used in nonvalvular atrial
fibrillation. Cryoprecipitate is a blood product used for replacement of von Willebrands factor, factor
XIII, fibrinogen, and fibronectin.
Ref:

Wachter RM, Goldman L, Hollander H (eds): Hospital Medicine, ed 2. Lippincott Williams & Wilkins, 2005, p 987. 2)
Burnett B, Kopecky S, Morton C, et al: Health Care Guideline: Antithrombotic Therapy Supplement, ed 10. Institute for
Clinical Systems Improvement, 2011.

Item 79
ANSWER:

Adults with a chronic cough lasting 2 months or longer who are nonsmokers and are not taking ACE
inhibitors should have plain radiographs to rule out specific causes prior to initiating empiric therapy (SOR
C). Any treatment should be targeted to the most likely cause. The three most common causes of chronic
cough in adults, other than ACE inhibitors, are gastroesophageal reflux disease, asthma, and upper airway
cough syndrome.
26

Patients who are taking an ACE inhibitor should be switched to another class of antihypertensive drugs.
Metoprolol does not cause a cough per se, although it may unmask preexisting asthma or COPD, resulting
in a cough. $-Blockers should not be discontinued abruptly, however. Formal spirometry and advanced
radiographic imaging have eventual roles in the evaluation of chronic cough but are expensive tests and
are not the best initial steps for evaluating a chronic cough.
Ref:

Benich JJ III, Carek PJ: Evaluation of the patient with chronic cough. Am Fam Physician 2011;84(8):887-892.

Item 80
ANSWER:

Chest compressions are recommended for a heart rate below 60 beats/min in a neonate.
Ref:

Raghuveer TS, Cox AJ: Neonatal resuscitation: An update. Am Fam Physician 2011;83(8):911-918.

Item 81
ANSWER:

At one point, the American Diabetes Association (ADA) recommended aspirin for all patients with diabetes
mellitus. They have since revised their guidelines and advise that aspirin not be used for primary
prevention of cardiovascular events unless a patients cardiovascular risk is >10% over 10 years. The
reason the ADA revised their guidelines on use of low-dose aspirin is because of the results from two
studies: the Prevention of Progression of Arterial Disease and Diabetes (POPADAD) study and the
Japanese Primary Prevention of Atherosclerosis with Aspirin for Diabetes (JPAD) study.
The POPADAD study compared aspirin versus placebo in patients with diabetes and found that death rates
from coronary heart disease and stroke were similar for the two groups, as were rates of nonfatal
myocardial infarction and nonfatal stroke. The JPAD study also compared aspirin vs. placebo in diabetic
patients, with similar rates of sudden death, nonfatal myocardial infarction, nonfatal stroke, unstable
angina, TIA, and peripheral vascular disease. Rates of fatal myocardial infarction and fatal stroke were
lower in the aspirin group.
Ref:

Jackson AN, Hume AL: Aspirin for CV preventionFor which patients? J Fam Pract 2011;60(9):518-523.

Item 82
ANSWER:

The flick sign has the highest sensitivity (93%) and specificity (96%) among the clinical findings of carpal
tunnel syndrome. This sign is defined as a history of shaking the hand or flicking the wrist in an attempt
to alleviate discomfort after being awakened with nighttime pain. Tinels sign and the Phalen maneuver
have a sensitivity of 36% and 57%, and a specificity of 75% and 58%, respectively. Thenar atrophy is
usually seen in severe and chronic cases of carpal tunnel syndrome and has a sensitivity of 16% and a
specificity of 90% (SOR B).
Ref:

LeBlanc KE, Cestia W: Carpal tunnel syndrome. Am Fam Physician 2011;83(8):952-958.

27

Item 83
ANSWER:

Evidence shows that $-blockers reduce mortality and hospitalization rates for patients with systolic heart
failure (SOR A). They should be started at a low dosage and increased to target dosages (SOR A).
$-Blockers should be considered even in patients with COPD and asthma, given their benefits. The benefit
of $-blockers is proportional to the degree of reduction in heart rate (SOR A).
Of the listed $-blockers, carvedilol has been shown to reduce the rates of death and hospitalization in heart
failure patients. Other $-blockers that have been established through randomized, controlled trials to
benefit heart failure patients are bisoprolol and metoprolol succinate. The effect of nebivolol on mortality
has not been adequately studied.
Ref:

Ong HT, Kow FP: Beta-blockers for heart failure: Why you should use them more. J Fam Pract 2011;60(8):472-477.

Item 84
ANSWER:

Cervical spondylotic myelopathy (CSM) is the most common cause of spinal cord dysfunction in the
elderly. Degenerative changes in the cervical spine, such as osteophyte formation, stiffened and
hypertrophied ligamentum flava, and spinal stenosis, can result in spinal cord compression. Symptoms
usually develop insidiously and may include neck stiffness, pain in the arm(s), tingling or numbness in the
hands, and weakness of the hands or legs. Flexion of the neck may produce a shock-like sensation down
the back, known as Lhermittes sign.
Sensory abnormalities may vary. Hyperreflexia is a characteristic physical finding. The gait may be stiff
or spastic, and atrophy of the intrinsic muscles of the hands is common. CSM can be differentiated from
amyotrophic lateral sclerosis (ALS) by the fasciculations and leg atrophy seen in ALS. Other conditions
that produce similar findings include multiple sclerosis and masses such as a metastatic tumor.
The primary diagnostic test is MRI of the cervical spine. Plain films are of little use as an initial diagnostic
procedure. Electromyography is usually not helpful, although it is occasionally needed to exclude
peripheral neuropathy.
Nonsurgical treatment such as cervical bracing may be used in mild cases of CSM, but once a frank
myelopathy occurs surgical intervention is the only option. Studies on bracing show variable results,
although it is reported that symptomatic patients may deteriorate neurologically during bracing.
Ref:

Young WF: Cervical spondylotic myelopathy: A common cause of spinal cord dysfunction in older persons. Am Fam
Physician 2000;62(5):1064-1070. 2) Longo DL, Fauci AS, Kasper DL, et al (eds): Harrisons Principles of Internal
Medicine, ed 18. McGraw-Hill, 2012, pp 3372-3373.

28

Item 85
ANSWER:

Normal newborns may lose up to 10% of their weight following birth, and should return to their birth
weight by the end of the first week of life. The steady addition of 47 oz of weight per week should result
in a doubling of birth weight by 46 months of age. During the second half of the first year of life an
addition of 35 oz/week is more the norm, resulting in a tripling of the birth weight by 1 year of age.
Breastfed infants tend to gain weight more quickly during the first 6 months of life, while formula-fed
infants do so from 612 months, with both groups having virtually equal weight gains by the end of the
first year.
Ref:

Hagan JF Jr, Shaw JS, Duncan PM (eds): Bright Futures Guidelines for Health Supervision of Infants, Children, and
Adolescents, ed 3. American Academy of Pediatrics, 2008, pp 125-126.

Item 86
ANSWER:

A Cochrane review found that melatonin was effective for reducing jet lag, especially when crossing 5 or
more time zones in an easterly direction. The drug can also be effective when crossing 24 time zones.
The most effective dosage seems to be 0.55 mg taken at bedtime starting on the day of arrival, with
higher doses being more effective. Taking it before departure does not help, and taking it earlier in the day
could make jet lag worse. There does not seem to be any benefit from taking melatonin prior to departure,
and melatonin is not recommended when flying westward.
Ref:

Herxheimer A, Petrie KJ: Melatonin for the prevention and treatment of jet lag. Cochrane Database Syst Rev
2002;(2):CD001520. 2) Bope ET, Kellerman RD (eds): Conns Current Therapy 2012. Elsevier Saunders, 2012, p 665.

Item 87
ANSWER:

Inhaled corticosteroids are the most potent and consistently effective long-term daily controller medications
for monotherapy of mild persistent asthma (SOR A). They can be successfully used in combination with
intermittent short-acting $-agonists. Oral systemic corticosteroids are recommended for moderate to severe
asthma exacerbations and usually for a very limited time period (SOR A). Daily long-acting $-agonists are
often used in combination with inhaled corticosteroids; however, long-acting $-agonists are not
recommended for use as daily monotherapy for long-term control of persistent asthma, or for intermittent
use (SOR A). Immunomodulators such as omalizumab prevent binding of immunoglobulin E to the
high-affinity receptors on basophils and mast cells. These are used as an additive therapy for patients age
12 years and older with severe persistent asthma, and are not recommended for routine use as
monotherapeutic agents.
Ref:

American Lung Association Asthma Clinical Research Centers, Peters SP, Anthonisen N, et al: Randomized comparison
of strategies for reducing treatment in mild persistent asthma. N Engl J Med 2007;356(20):20272039. 2) Expert Panel
Report 3: Guidelines for the Diagnosis and Management of Asthma. National Asthma Education and Prevention Program,
NIH pub no 07-4051, 2007. 3) Elward KS, Pollart SM: Medical therapy for asthma: Updates from the NAEPP guidelines.
Am Fam Physician 2010;82(10):1242-1251.

29

Item 88
ANSWER:

Celiac sprue (gluten-sensitive enteropathy) classically presents as a malabsorption syndrome associated


with dermatitis herpetiformis. This dermatitis usually appears as excoriated papules, as it is extremely
pruritic. The rash may be misdiagnosed as atypical psoriasis or nonspecific dermatitis. With the
development and use of better diagnostic tests, it now appears that this disorder has been underdiagnosed.
Symptoms include fatigue, weight loss, diarrhea, abdominal pain, anemia, bone pain, aphthous ulcers,
stomatitis, infertility, impotence, alopecia areata, dental enamel defects, seizures, ataxia, and dermatitis.
Serologic tests are now available to aid in confirming the diagnosis of celiac sprue, including IgA
antigliadin antibody, IgG antigliadin antibody, IgA antiendomysial antibody, and IgA antitransglutaminase.
Cystic fibrosis, Crohns disease, and anorexia nervosa can cause weight loss but not dermatitis. Sprue
affects the small intestine; a biopsy of the colon would be inappropriate given this presentation.
Ref:

Nelsen DA Jr: Gluten-sensitive enteropathy (celiac disease): More common than you think. Am Fam Physician
2002;66(12):2259-2266. 2) Rossi T: Celiac disease. Adolesc Med Clin 2004;15(1):91-103. 3) Longo DL, Fauci AS,
Kasper DL, et al (eds): Harrisons Principles of Internal Medicine, ed 18. McGraw-Hill, 2012, pp 2469-2471.

Item 89
ANSWER:

The patient described has a history compatible with gallbladder disease. In a patient with such a typical
history, abdominal ultrasonography is likely to show gallstones and thus provide support for the diagnosis.
Serum bilirubin and AST levels are usually normal except at the time of an attack. A HIDA scan may be
useful if performed during an attack, since the scan assesses the patency of the cystic duct. A plain
abdominal film will detect only 10%15% of cases of cholelithiasis.
Ref:

Friedman LS, Brandt LJ (eds): Sleisenger & Fordtrans Gastrointestinal and Liver Disease, ed 9. Saunders, 2010, pp
1105-1113. 2) Townsend CM Jr, Beauchamp RD, Evers BM, et al: Sabiston Textbook of Surgery: The Biological Basis
of Modern Surgical Practice, ed 19. Saunders, 2012, pp 1485-1487.

Item 90
ANSWER:

In preschool-age children, lower respiratory infections such as pneumonia are most commonly viral
illnesses. Antibiotics may be withheld in young children who are mildly ill and are suspected of having a
viral disease, but antibiotic therapy should be started if their clinical status worsens. In the preschool-age
child with pneumonia, amoxicillin remains the first-line antibiotic of choice, as it provides coverage for
Streptococcus pneumoniae and Haemophilus influenzae, which are the predominant bacterial causes of
pneumonia in this age group. The pharmacokinetics of azithromycin do not preclude its use in children,
but it is not the first-line choice for this patient.
Viruses are also the most frequent cause of pneumonia in the older child, although after the age of 5 years
atypical pneumonia becomes more common. This requires antibiotic coverage for organisms such as
Mycoplasma. For these patients, empiric treatment with a macrolide antibiotic such as azithromycin is
appropriate.

30

Ref:

Chang AB, Chang CC, OGrady K, Torzillo PJ: Lower respiratory tract infections. Pediatr Clin North Am
2009;56(6):1303-1321. 2) Ebell MH: Clinical diagnosis of pneumonia in children. Am Fam Physician 2010;82(2):192-193.
3) Bradley JS, Byington CL, Shah SS, et al: The management of community-acquired pneumonia in infants and children
older than 3 months of age: Clinical practice guidelines by the Pediatric Infectious Diseases Society and the Infectious
Diseases Society of America. Clin Infect Dis 2011;53(7):e25-e76.

Item 91
ANSWER:

ACE inhibitors improve the quality of life and the prognosis for patients with myocarditis-induced dilated
cardiomyopathy, just as they do for other patients with heart failure. Neither antiviral therapy nor
immunosuppression has been shown to improve this type of cardiomyopathy when tested in controlled
trials. NSAIDs actually increase mortality by worsening sodium retention.
Ref:

Cooper LT Jr: Myocarditis. N Engl J Med 2009;360(15):1526-1538.

Item 92
ANSWER:

Treatment of enterohemorrhagic Escherichia coli infection consists of supportive measures only.


Antibiotics are contraindicated because they can trigger the release of Shiga toxins, which may lead to
hemolytic-uremic syndrome in children.
Ref:

Mandell GL, Bennett JE, Dolin R (eds): Mandell, Douglas, and Bennetts Principles and Practice of Infectious Diseases,
ed 7. Churchill Livingstone, 2009, p 2825.

Item 93
ANSWER:

Repair of symptomatic pectus excavatum should be postponed until adolescence, if possible, as this
approach allows for completion of growth and reduces the chance of recurrence. Younger children with
severe cardiopulmonary problems may also be candidates for surgery, but repair at too early an age can
result in improper growth of the chest wall and increases the risk of recurrence of the deformity. Adult
repair is also feasible.
Ref:

Jaroszewski D, Notrica D, McMahon L, et al: Current management of pectus excavatum: A review and update of therapy
and treatment recommendations. J Am Board Fam Med 2010;23(2):230-239.

Item 94
ANSWER:

More than 50% of U.S. hospitals have palliative care programs, which focus on pain and symptom
management. These programs decrease both overall hospital costs and ICU use. Because palliative care
requires a team approach, the number of nonphysician personnel is not decreased. The length of patient
hospital stays is also not decreased (level of evidence 2, SOR A).
Ref:

Penrod JD, Deb P, Dellenbaugh C, et al: Hospital-based palliative care consultation: Effects on hospital cost. J Palliat Med
2010;13(8):973-979.

31

Item 95
ANSWER:

All patients who present with palpitations should be evaluated for a cardiac cause, since this is the etiology
in 43% of cases. A standard 12-lead EKG is the initial test of choice and, along with a history and physical
examination, can determine the cause in 40% of cases. A normal resting EKG does not exclude a cardiac
arrhythmia. Therefore, if the EKG is normal, palpitations of suspected arrhythmic etiology may require
further investigation with ambulatory EKG monitoring.
Echocardiography is helpful in evaluating patients for structural heart disease and should be performed
when the initial history, physical examination, and EKG are unrevealing, or in patients with a history of
cardiac disease or more complex signs and symptoms. This patients family history, along with the fact
that she takes no medications, suggests the possibility of familial long QT syndrome, which often can be
diagnosed from a resting EKG. Diagnosing long QT syndrome is important, since it is associated with an
increased risk of sudden cardiac death. Based on the patients clinical presentation and evaluation, hospital
admission is not warranted prior to obtaining a standard 12-lead EKG.
Ref:

Wexler RK, Pleister A, Raman S: Outpatient approach to palpitations. Am Fam Physician 2011;84(1):63-69.

Item 96
ANSWER:

Medicare has an established definition of what constitutes a homebound patient. The definition includes
patients who require the use of a cane or other supportive device in order to leave the home (not just
occasional use) or require the help of another person to leave the home. Participation in a state-licensed
adult day care program or regularly attending religious services does not disqualify a person from being
considered confined to the home.
Ref:

Unwin BK, Tatum PE III: House calls. Am Fam Physician 2011;83(8):925-931.

Item 97
ANSWER:

Breast milk is considered potentially infectious in patients with HIV infection, along with vaginal
secretions, semen, and blood. Contact with saliva, sweat, urine, or feces does not require postexposure
prophylaxis.
Ref:

Tolle MA, Schwarzwald HL: Postexposure prophylaxis against human immunodeficiency virus. Am Fam Physician
2010;82(2):161-166.

32

Item 98
ANSWER:

Placenta previa is incidentally found on approximately 4% of sonograms performed between 20 and 24


weeks gestation. It often will resolve, and the incidence at term is approximately 0.4%. Symptomatic
placenta previa usually manifests as painless bleeding in the late second or third trimester. It can be painful
bleeding if it is associated with labor or abruption. Most patients with symptomatic placenta previa will
be admitted to the hospital for evaluation. Most neonatal morbidity and mortality associated with placenta
previa is due to the risks associated with preterm birth.
Corticosteroids should be given to women who present with bleeding from a placenta previa between 24
and 34 weeks gestation (SOR A). Tocolytic agents such as magnesium or calcium channel blockers would
be appropriate in patients who have vaginal bleeding associated with preterm contractions.
The goal with tocolytic treatment would be to prolong the pregnancy until fetal lung maturity is achieved.
This patient is not having preterm contractions so tocolytics would not be appropriate. The fetal heart rate
is stable and the mother is hemodynamically stable, so there is no indication for an urgent cesarean section.
Antibiotics do not have a role in the management of symptomatic placenta previa.
Ref:

Sakornbut E, Leeman L, Fontaine P: Late pregnancy bleeding. Am Fam Physician 2007;75(8):1199-1206.

Item 99
ANSWER:

The response to epinephrine may be limited in patients with anaphylaxis who have been taking $-blockers.
Such individuals may have persistent hypotension, bradycardia, and prolonged symptoms. Since glucagon
exerts positive inotropic and chronotropic effects on the heart without depending on catecholamines, an
intravenous bolus followed by an infusion would be a good choice to treat the refractory hypotension.
The use of corticosteroids in this setting is common, but their effectiveness has not been established. Their
benefit is not realized for at least 6 hours, however, so they may aid in the prevention of recurrent
anaphylaxis. Diphenhydramine sometimes provides dramatic symptom relief, but it would not improve the
hypotension. $-Agonists such as albuterol and aminophylline can be used for bronchospasm, but are not
helpful for hypotension.
Ref:

Tang AW: A practical guide to anaphylaxis. Am Fam Physician 2003;68(7):1325-1332. 2) Thomas M, Crawford I:
Glucagon infusion in refractory anaphylactic shock in patients on beta-blockers. Emerg Med J 2005;22(4):272-273. 3) Marx
JA (ed): Rosens Emergency Medicine: Concepts and Clinical Practice, ed 7. Mosby Elsevier, 2010, pp 1521-1525.

33

Item 100
ANSWER:

The U.S. Preventive Services Task Force has concluded that current evidence is insufficient to assess the
balance of benefits and harms of whole-body skin examination by a primary care physician or by patient
skin self-examination for the early detection of cutaneous melanoma, basal cell cancer, or squamous cell
skin cancer in the adult general population. Due to the lack of studies, the evidence is insufficient to
determine whether early detection of skin cancer reduces mortality or morbidity from skin cancer. The
same is true regarding the magnitude of harms from screening for skin cancer. Benefits from screening
are uncertain, even in high-risk patients.
Ref:

US Preventive Services Task Force: Screening for skin cancer: Recommendation statement. Am Fam Physician
2010;81(12):1433-1434.

Item 101
ANSWER:

This patient has several red flags that require complete colon evaluation with endoscopy: age >50, a
change in stool caliber, and obstructive symptoms. Other red flags include heme-positive stools, anemia
consistent with iron deficiency, and rectal bleeding. Malignancy should be eliminated as a possible
diagnosis prior to initiating any treatment. Biofeedback training is used to manage pelvic floor dysfunction
caused by incoordination of pelvic floor muscles during attempted evacuation. Common symptoms include
prolonged or excessive straining, soft stools that are difficult to pass, and rectal discomfort. The other
options are appropriate management strategies once malignancy has been eliminated as a possibility.
Ref:

Jamshed N, Lee Z, Olden KW: Diagnostic approach to chronic constipation in adults. Am Fam Physician
2011;84(3):299-306.

Item 102
ANSWER:

Onychomycosis is a difficult condition to treat successfully. If symptoms are minimal, treatment is often
deferred. Cellulitis of the involved extremity may be related to the onychomycosis and is an accepted
reason to consider eradication treatment. Oral terbinafine is the best treatment in terms of cure rate and
tolerability (SOR A). Significant liver disease is a contraindication.
Itraconazole is less effective and more toxic, and griseofulvin is significantly less effective. Topical
ciclopirox lacquer is also less effective than terbinafine, although it eliminates the risk for systemic
toxicity. Fluconazole is not indicated for onychomycosis.
Ref:

Hinojosa JR, Hitchcock K, Rodriguez JE: Clinical inquiries. Which oral antifungal is best for toenail onychomycosis? J
Fam Pract 2007;56(7):581-582.

34

Item 103
ANSWER:

Providing quality health care to individuals from diverse sociocultural backgrounds requires effective
communication. Low health literacy in almost half of the U.S. population makes communication more
difficult. When a language barrier exists it is better to have a professional interpreter than a family
member, and children should be used as interpreters only in cases of emergency when no other source is
available.
The typical approach to medical care in the United States assumes that patients want to make their own
decisions based on guidance from their health care providers. However, there are cultures in which patient
autonomy is not the norm. There may be a specific authority figure in the family that is regarded as the
decision maker.
Effective communication also involves knowledge of communication styles within various cultures.
Nonverbal communication can be through touch, eye contact, and personal space. For example, there are
cultures in which direct eye contact is avoided, but in other cultures it is considered a sign of respect.
Ref:

Blackhall LJ, Murphy ST, Frank G, et al: Ethnicity and attitudes toward patient autonomy. JAMA 1995;274(10):820-825.
2) Green AR, Betancourt JR, Carrillo JE: Integrating social factors into cross-cultural medical education. Acad Med
2002;77(3):193-197. 3) Juckett G: Cross-cultural medicine. Am Fam Physician 2005;72(11):2267-2274. 4) Schyve PM:
Language differences as a barrier to quality and safety in health care: The Joint Commission perspective. J Gen Intern Med
2007;22(Suppl 2):360-361. 5) Schenker Y, Lo B, Ettinger KM, Fernandez A: Navigating language barriers under difficult
circumstances. Ann Intern Med 2008;149(4):264-269.

Item 104
ANSWER:

Any patient with risk factors for infection who presents with acute joint swelling, pain, erythema, warmth,
and joint immobility should be evaluated for septic arthritis. Risk factors for septic arthritis in this patient
include a cutaneous ulcer and diabetes mellitus. Serum markers such as the WBC count, erythrocyte
sedimentation rate, and C-reactive protein levels are often used to determine the presence of infection or
inflammatory response. However, patients with confirmed septic arthritis may have normal erythrocyte
sedimentation rates and C-reactive protein levels.
Because the clinical presentation of septic arthritis may overlap with that of other causes of acute arthritis,
arthrocentesis is needed to differentiate between the various causes and, in the case of septic arthritis, to
identify the causative agent and determine appropriate therapy. No findings on imaging studies are
pathognomonic for septic arthritis. Antinuclear antibody studies may be indicated later in the course of
management if synovial fluid analysis is not consistent with infection, and if synovial fluid cultures are
negative.
Ref:

Horowitz DL, Katzap E, Horowitz S, Barilla-LaBarca ML: Approach to septic arthritis. Am Fam Physician
2011;84(6):653-660.

35

Item 105
ANSWER:

Norepinephrine is the recommended first-line vasopressor agent to correct hypotension in patients with
sepsis (SOR A). Vasopressor therapy is required to sustain life and maintain perfusion in the face of
life-threatening hypotension, even when hypovolemia has not yet been resolved. Maintaining a mean
arterial pressure of at least 65 mm Hg is critical for tissue perfusion. Dopamine is recommended as an
alternate first-line agent to elevate arterial pressure, but it is less potent compared to norepinephrine.
Dobutamine is recommended as the first-line agent for managing hypotension in cardiogenic shock.
Phenylephrine is recommended as the second-line agent for managing hypotension in patients with septic
shock who also have tachycardia or dysrhythmias. Albumin and epinephrine are not recommended as
first-line agents for managing hypotension in patients with sepsis.
Ref:

Dellinger RP, Levy MM, Carlet JM, et al: Surviving Sepsis Campaign: International guidelines for management of severe
sepsis and septic shock: 2008. Crit Care Med 2008;36(1):296-327.

Item 106
ANSWER:

A prolonged grief reaction, postpartum depression, and iron deficiency anemia could all cause fatigue,
apathy, and decreased libido, but none of these conditions is characterized by failure to lactate and
amenorrhea. This patient most likely has postpartum pituitary necrosis (Sheehans syndrome), a
complication of childbirth in which hemorrhagic shock leads to pituitary necrosis. The syndrome is caused
by the lack of hormonal influence from the anterior pituitary gland on other endocrine glands, resulting
in failure to lactate, breast atrophy, mental apathy, low blood pressure, absence or deficiency of sweating,
loss of secondary hair characteristics and libido, and loss of ovarian function, resulting in amenorrhea.
Ref:

Master-Hunter T, Heiman DL: Amenorrhea: Evaluation and treatment. Am Fam Physician 2006;73(8):1374-1382. 2) Fritz
MA, Speroff L: Clinical Gynecologic Endocrinology and Infertility, ed 8. Lippincott Williams & Wilkins, 2011, pp
483-484.

Item 107
ANSWER:

This patient should have an endometrial biopsy (SOR C). Approximately 7% of postmenopausal women
with benign endometrial cells on a Papanicolaou smear will have significant endometrial pathology. None
of the other options listed evaluate the endometrium for pathology. An asymptomatic premenopausal
woman with benign endometrial cells would not need an endometrial evaluation because underlying
endometrial pathology is rare in this group.
Ref:

Apgar BS, Kittendorf AL, Bettcher CM, et al: Update on ASCCP consensus guidelines for abnormal cervical screening
tests and cervical histology. Am Fam Physician 2009;80(2):147-155. 2) Buchanan EM, Weinstein LC, Hillson C:
Endometrial cancer. Am Fam Physician 2009;80(10):1075-1080, 1087-1088.

36

Item 108
ANSWER:

Repaglinide is a non-sulfonylurea agent that interacts with a different portion of the sulfonylurea receptor
to stimulate insulin secretion. It has a relatively short duration of action, and while it may cause
hypoglycemia this is less likely than with a sulfonylurea agent. Pioglitazone reduces insulin resistance and
has no hypoglycemic effect. Acarbose delays absorption of carbohydrates such as starch, sucrose, and
maltose, but does not affect the absorption of glucose and other monosaccharides. Sitagliptin inhibits the
enzyme responsible for the breakdown of the naturally occurring incretins, and its major advantage is the
absence of side effects. Exenatide stimulates insulin secretion in a glucose-dependent fashion, inhibits
glucagon secretion, slows gastric emptying, and may have a central satiety effect. It does not cause
hypoglycemia when used as monotherapy, but may increase the risk when used with an insulin
secretagogue such as glyburide or glipizide.
Ref:

Goldman L, Schafer AI (eds): Goldmans Cecil Medicine, ed 24. Elsevier Saunders, 2011, pp e100-e104.

Item 109
ANSWER:

Although the registration of death is a state function and the details may vary based on the laws and
regulations of each state, recorded data is contractually shared with the National Vital Statistics System.
To ensure consistency of reporting, the National Center for Health Certificates coordinates collection of
the data points by providing a standard form which most state certificates are modeled from. The standard
format includes a section titled Cause of Death, which is subdivided into two parts. In part 1, the
immediate cause of death is to be recorded on the top line (labeled a). This is defined as the final
disease, injury, or complication directly causing the death, and the directions clearly state that terminal
events such as cardiac arrest, respiratory arrest, or ventricular fibrillation are not to be entered without
showing the etiology.
Additional lines are provided to list conditions leading to the cause of death, including a final line for
entering the disease or injury that initiated the process leading to death. In this case, the proximate cause
of death was the upper gastrointestinal hemorrhage. The source of the bleeding was most likely from
esophageal varices resulting from hepatic cirrhosis, so those conditions should be entered respectively in
the next two lines. The appropriate entry for the final line in part 1 would be chronic alcoholism.
Space is provided in part 2 to include significant conditions contributing to death, such as other chronic
illness and tobacco use.
Ref:

Physicians Handbook on Medical Certification of Death. Centers for Disease Control and Prevention, National Center for
Health and Statistics, DHHS pub no (PHS) 2003-1108, 2003, pp 20-23. 2) Hanzlick R: Writing cause of death
statementsBasic principles. National Association of Medical Examiners, 2005.

37

Item 110
ANSWER:

Patients with a life expectancy of 6 months or less are eligible for the Medicare hospice benefit. This
benefit allows patients to receive hospice care in either the home or hospital setting. In addition to patients
with terminal cancer, patients with end-stage cardiac, pulmonary, and chronic debilitating diseases are
eligible. Approximately two-thirds of patients enrolled in hospice die from noncancer-related diagnoses,
and approximately 60% of Medicare patients are not enrolled in hospice at the time of their death.
Ref:

Kaprow MG: Use of hospice care for patients without cancer. Am Fam Physician 2010;82(10):1196.

Item 111
ANSWER:

Ipratropium bromide is an anticholinergic agent. When nebulized it can sometimes cause inadvertent ocular
effects. Blurred vision and pupil inequality may occur. Ipratropium has been shown to decrease the rate
of hospital admissions in severe asthmatic attacks.
Corticosteroids may elevate glucose levels in diabetic patients. The onset would be more gradual, however.
Adrenergic agents used for acute asthma do not commonly produce adverse ocular effects (level of
evidence 3).
Ref:

Lazarus SC: Emergency treatment of asthma. N Engl J Med 2010;363(8):755-764.

Item 112
ANSWER:

While the U.S. Preventive Services Task Force (USPSTF) recommends against screening for prostate
cancer using prostate-specific antigen testing, other screening methods have not been evaluated in
controlled studies. For men who have smoked, one-time ultrasonography is recommended as a screen for
aortic aneurysm between the ages of 65 and 75. The USPSTF has no recommendation for men who have
never smoked.
The USPSTF states that no evidence supports routine colorectal cancer screening in patients age 7685,
but that there may be some individuals with specific considerations for whom colorectal cancer screening
would be recommended.
At present, there is no evidence to support screening of older adults for dementia, but it is recommended
that all adults be screened for depression when staff support is in place to ensure adequate diagnosis,
treatment, and follow-up. In most instances, the elderly population will present to a primary care provider
with somatic complaints (level of evidence 1b).
Ref:

Screening for dementia: Recommendations and Rationale. US Preventive Services Task Force, 2003. 2) Screening for
abdominal aortic aneurysm: Recommendation statement. US Preventive Services Task Force, 2005. 3) Screening for
colorectal cancer: Recommendation statement. US Preventive Services Task Force, 2008. 4) Screening for depression in
adults: Recommendation statement. US Preventive Services Task Force, 2009. 5) Screening for prostate cancer: US
Preventive Services Task Force recommendation statement. US Preventive Services Task Force, 2012.

38

Item 113
ANSWER:

Amyloidosis is defined as the extracellular deposition of the fibrous protein amyloid at one or more sites.
It may remain undiagnosed for years. Features that should alert the clinician to the diagnosis of primary
amyloidosis include unexplained proteinuria, peripheral neuropathy, enlargement of the tongue,
cardiomegaly, intestinal malabsorption, bilateral carpal tunnel syndrome, or orthostatic hypotension.
Amyloidosis occurs both as a primary idiopathic disorder and in association with other diseases such as
multiple myeloma.
Ref:

Klippel JH, Stone JH, Crofford LJ, et al (eds): Primer on the Rheumatic Diseases, ed 13. Springer, 2008, pp 533-541.
2) Longo DL, Fauci AS, Kasper DL, et al (eds): Harrisons Principles of Internal Medicine, ed 18. McGraw-Hill, 2012,
pp 945-950.

Item 114
ANSWER:

Most uninsured people in the United States are members of a family with at least one working adult. Most
uninsured people who are employed work for small companies or work part-time. Most uninsured people
who work part-time with incomes below the poverty line are not eligible for Medicaid. On average,
uninsured people have less access to care and have poorer health outcomes.
Ref:

Tunzi M: The uninsured. Am Fam Physician 2004;69(6):1357-1360. 2) The Henry J. Kaiser Family Foundation: The
Uninsured: A Primer. Key Facts About Americans Without Health Insurance. October 2011, pp 6-8.

Item 115
ANSWER:

If semen analysis suggests hypogonadism (e.g., severe oligospermia or azoospermia), it is important to


distinguish between primary and secondary causes (SOR C). Evaluation of morning FSH and total serum
testosterone levels can help make this determination. Low testosterone levels correlate with hypogonadism.
High levels of FSH in the presence of low testosterone levels correlate with primary hypogonadism (SOR
B). Low levels of both hormones suggest secondary hypogonadism (SOR B). High testosterone levels are
unlikely to be associated with hypogonadism.
Ref:

Jose-Miller AB, Boyden JW, Frey KA: Infertility. Am Fam Physician 2007;75(6):849-856. 2) American Academy of
Family Physicians: Information from your family doctor. Infertility: What you should know. Am Fam Physician
2007;75(6):857-858.

39

Item 116
ANSWER:

Several medications from different classes are recommended as first-line abortive therapies to treat acute
migraine. Because relatively few trials have directly compared the different medication classes, there are
no definitive algorithms as to which class works best. NSAIDs and acetaminophen/aspirin/caffeine are
recommended as first-line therapies and can be obtained over the counter (SOR A). Triptans are effective
and safe for treatment of acute migraine and are recommended as first-line therapy (SOR A) but require
a prescription. Opiates and barbiturates are not recommended because of their potential for abuse (SOR
C). Acetaminophen alone is not effective, and the same is true of oral corticosteroids.
Ref:

Silberstein SD: Practice parameter: Evidence-based guidelines for migraine headache (an evidence-based review): Report
of the Quality Standards Subcommittee of the American Academy of Neurology. Neurology 2000;55(6):754-762. 2)
Tfelt-Hansen P, De Vries P, Saxena PR: Triptans in migraine: A comparative review of pharmacology, pharmacokinetics
and efficacy. Drugs 2000;60(6):1259-1287. 3) Ferrari MD, Roon KI, Lipton RB, Goadsby PJ: Oral triptans (serotonin
5-HT(1B/1D) agonists) in acute migraine treatment: A meta-analysis of 53 trials. Lancet 2001;358(9294):1668-1675. 4)
McCrory DC, Gray RN: Oral sumatriptan for acute migraine. Cochrane Database Syst Rev 2003;(3):CD002915. 5)
Gilmore B, Michael M: Treatment of acute migraine headache. Am Fam Physician 2011;83(3):271-280.

Item 117
ANSWER:

Proton pump inhibitors (PPIs) have a powerful effect on inhibiting the production of acid in the stomach.
This dramatically reduces symptoms of acid-mediated gastritis, peptic ulcer disease, and gastroesophageal
reflux. However, a significant reduction in stomach acidity may cause unintended consequences involving
processes that are physiologically dependent on low pH in the gastrointestinal tract. These theoretical risks
include decreased levels of vitamin B12, iron, and/or magnesium; decreased bone density; an increase in
gut infections or pneumonia; an increase in gastrointestinal neoplasms; and changes in absorption of other
medications.
The evidence has been conflicting on some of these risks. Currently, consensus is emerging that chronic
use of PPIs increases the risk for pneumonia and gut infections, primarily Clostridium difficile colitis (SOR
B). PPIs may also decrease bone density in subsets of patients. These risks need to be weighed against the
benefits that these medicines provide before prescribing them on a long-term basis.
Ref:

Sheen E, Triadafilopoulos G: Adverse effects of long-term proton pump inhibitor therapy. Dig Dis Sci 2011;56(4):931-950.
2) Eom CS, Jeon CY, Lim JW, et al: Use of acid-suppressive drugs and risk of pneumonia: A systematic review and
meta-analysis. CMAJ 2011;183(3):310-319. 3) Kwok CS, Arthur AK, Anibueze CI, et al: Risk of Clostridium difficile
infection with acid suppressing drugs and antibiotics: Meta-analysis. Am J Gastroenterol 2012;107(7):1011-1019.

Item 118
ANSWER:

The diagnosis of idiopathic scoliosis is based on a coronal plane curvature >10. It is a diagnosis of
exclusion after congenital, neuromuscular, and myopathic diseases and conditions have been ruled out.
Adolescent scoliosis is most common, and occurs in about 2%3% of adolescents. More marked curvature
(>30) occurs in about 0.3% of adolescents, as measured on posterior-anterior and lateral radiographs
using the Cobb method. For mild degrees of curvature there is an even distribution between girls and boys,
but girls have a tenfold greater risk for more severe curvature.

40

Screening for scoliosis in the asymptomatic adolescent is controversial; the U.S. Preventive Services Task
Force recommends against routine screening in its most recent update in 2004 (D recommendation).
However, if idiopathic scoliosis is discovered incidentally or when the adolescent or parent expresses
concern about scoliosis, options for further evaluation and treatment include observation for curvatures
of less than 20 and consideration for bracing and/or surgery for more severe curvatures. The risk of
progression depends on the amount of growth remaining, the magnitude of the curve, and the patients
gender.
Ref:

Screening for idiopathic scoliosis in adolescents: Brief evidence update. US Preventive Services Task Force, 2004. 2)
Kliegman RM, Stanton BF, Geme JW III, et al (eds): Nelson Textbook of Pediatrics, ed 19. Elsevier Saunders, 2011, p
2367.

Item 119
ANSWER:

This patient suffers from chronic low back pain, defined as pain, muscle tension, or stiffness localized
below the costal margin and above the inferior gluteal folds, with or without sciatica, that has persisted for
12 weeks or more and is not attributed to a recognizable pathology. Among all the listed treatment options
for chronic low back pain, only back exercises are given the beneficial recommendation in a systematic
review. Acupuncture and spinal manipulation are in the likely to be beneficial category. Back school and
epidural corticosteroid injections are of unknown effectiveness.
Ref:

Chou R: Low back pain (chronic). Am Fam Physician 2011;84(4):437-438.

Item 120
ANSWER:

Myasthenia gravis is a neuromuscular illness with an underlying immune-related cause. Corticosteroids


and anticholinesterase medications such as oral pyridostigmine can be helpful, but thymectomy may be
appropriate for patients with generalized disease not responding to medication. Thymectomy increases the
remission rate and improves the clinical course.
Ref:

Goldman L, Schafer AI (eds): Goldmans Cecil Medicine, ed 24. Elsevier Saunders, 2011, pp 2418-2422.

Item 121
ANSWER:

According to the 2010 American Heart Association scientific statement regarding acute heart failure
syndrome, levels of natriuretic peptides such as BNP lack the specificity necessary to function as absolute
indicators of acute heart failure syndrome even when they exceed established thresholds for the diagnosis.
BNP levels vary with age, sex, body habitus, renal function, and abruptness of symptom onset.
Elevated BNP levels also have been associated with renal failure (because of reduced clearance),
pulmonary embolism, pulmonary hypertension, and chronic hypoxia. BNP measures are not a substitute
for a comprehensive assessment for signs and symptoms of heart failure, and a laboratory test by itself
cannot be used to determine the diagnosis or management of heart failure. Clinical evaluation and
follow-up are essential to assure proper care for patients with heart failure or any other cardiac problem.

41

Ref:

Doust J, Lehman R, Glasziou P: The role of BNP testing in heart failure. Am Fam Physician 2006;74(11):1893-1898. 2)
Weintraub NL, Collins SP, Pang PS, et al: Acute heart failure syndromes: Emergency department presentation, treatment,
and disposition: Current approaches and future aims: A scientific statement from the American Heart Association.
Circulation 2010;122(19):1975-1996.

Item 122
ANSWER:

Erythema, redness, and pain in the first metatarsal-phalangeal joint are typical symptoms of gout. The uric
acid level can be normal at various times in gout. Acute synovitis is occasionally caused by apatite
deposition disease, but it is usually associated with long-standing osteoarthritis, and the joints involved are
most commonly the shoulder, hip, and knee. Mortons neuroma is an entrapment neuropathy of the
interdigital nerve, usually occurring between the third and fourth toes, not associated with erythema and
redness. Acute arthritis in systemic lupus erythematosus typically involves the wrists, the small joints of
the hands, and the knees.
Ref:

Klippel JH, Stone JH, Crofford LJ, et al (eds): Primer on the Rheumatic Diseases, ed 13. Springer, 2008, pp 83-84,
241-249, 267-269, 307-308.

Item 123
ANSWER:

Studies have shown that the use of intravenous tissue plasminogen activator offers sustained patient benefit
at 6 and 12 months if given within 3 hours of symptom onset (SOR B). All other listed interventions have
not been shown to be efficacious.
Ref:

Wahlgren N, Ahmed N, Dvalos A, et al: Thrombolysis with alteplase for acute ischaemic stroke in the Safe
Implementation of Thrombolysis in Stroke-Monitoring Study (SITS-MOST): An observational study. Lancet
2007;369(9558):275-282. 2) Hacke W, Kaste M, Bluhmki E, et al: Thrombolysis with alteplase 3 to 4.5 hours after acute
ischemic stroke. N Engl J Med 2008;359(13):1317-1329. 3) Ferri FF (ed): Ferris Clinical Advisor 2012: 5 Books in 1.
Elsevier Mosby, 2012, pp 969-971.

Item 124
ANSWER:

Term infants with an onset of jaundice before 24 hours of age, jaundice persisting beyond 3 weeks of age,
or a bilirubin level requiring intensive phototherapy should not be considered healthy, and require further
evaluation. A 2-day-old term infant with a total bilirubin of 10 mg/dL may be followed expectantly.
Ref:

Kliegman RM, Stanton BF, Geme JW III, et al (eds): Nelson Textbook of Pediatrics, ed 19. Elsevier Saunders, 2011, pp
603-608.

42

Item 125
ANSWER:

Vitamin B12 and folate deficiencies typically cause macrocytic anemias. When the serum vitamin B12 level
is borderline low, an elevated methylmalonic acid level can be used to confirm a vitamin B12 deficiency.
An elevated homocysteine level plays a similar role for folate deficiency anemia. Hemolysis can be
associated with an elevated LDH level, and serum ferritin is useful for diagnosing iron deficiency anemia.
Ref:

Bross MH, Soch K, Smith-Knuppel T: Anemia in older persons. Am Fam Physician 2010;82(5):480-487.

Item 126
ANSWER:

Noninferiority trials compare an active control group with a new therapy. The use of a placebo group
would be unethical, since the present therapy is either lifesaving or prevents serious injury. The new
therapy may prove superior to or slightly less effective than the standard therapy.
Ref:

Non-inferiority trials. Med Lett Drugs Ther 2011;53:1.

Item 127
ANSWER:

Much of the psychological distress caused by the diagnosis of mitral valve prolapse is related to a lack of
information and a fear of heart disease, which may be reinforced by the death of a friend or relative. A
clear explanation of mitral valve prolapse, along with printed material, is a powerful aid in relieving the
patients emotional distress. The American Heart Association publishes a helpful booklet about this
condition which can be given to these patients. It is important to avoid reinforcing illness behavior with
unnecessary testing, medications, or referrals to specialists.
Ref:

Bonow RO, Mann DL, Zipes DP, Libby P (eds): Braunwalds Heart Disease: A Textbook of Cardiovascular Medicine, ed
9. Elsevier Saunders, 2011, pp 1510-1514. 2) Goldman L, Schafer AI (eds): Goldmans Cecil Medicine, ed 24. Elsevier
Saunders, 2011, pp 460-461.

Item 128
ANSWER:

All of the treatment options listed may improve the patients depression, but it is unnecessary to add a
second agent until the initial drug is at the maximum recommended dosage. Citalopram can be increased
to a dosage of 40 mg/day.
Ref:

Little A: Treatment-resistant depression. Am Fam Physician 2009;80(2):167-172.

43

Item 129
ANSWER:

Patients with rheumatoid arthritis (RA) are at increased risk for various extra-articular manifestations of
the inflammatory disease, as well as side effects of the medications used to manage it. The leading cause
of death in RA patients is cardiovascular, related to accelerated atherosclerosis (SOR C). Patients with RA
should be screened for cardiovascular risk factors and managed appropriately to lower their risk.
Patients with RA are also at increased risk for other problems that are not leading causes of mortality.
Their risk for infection is increased, which can be related to either the RA itself or to the use of
immunosuppressive agents. Patients with RA also have a twofold increase in their risk for lymphoma. This
is independent of whether or not they are on immunosuppressive agents. Their risk for lung cancer related
to interstitial lung disease is also increased, and smoking increases this risk further.
Ref:

Friedewald VE, Ganz P, Kremer JM, et al: AJC editors consensus: Rheumatoid arthritis and atherosclerotic cardiovascular
disease. Am J Cardiol 2010;106(3):442-447. 2) Wasserman AM: Diagnosis and management of rheumatoid arthritis. Am
Fam Physician 2011;84(11):1245-1252.

Item 130
ANSWER:

This patient likely has meningitis due to Neisseria meningitidis. Ceftriaxone is recommended as first-line
therapy and should not be delayed once the diagnosis is suspected (SOR B). Ciprofloxacin and rifampin
are not recommended as first-line therapy for infected individuals, but are recommended as prophylaxis
for close contacts (SOR B). Doxycycline and amoxicillin are not proven to be effective for treatment or
prophylaxis.
Ref:

Bamberger DM: Diagnosis, initial management, and prevention of meningitis. Am Fam Physician 2010;82(12):1491-1498.

Item 131
ANSWER:

The 2010 American Heart Association guidelines for resuscitation emphasize the importance of chest
compression in CPR. Compression-Airway-Breathing (C-A-B) is now recommended over AirwayBreathing-Compression (A-B-C). Individualization of this sequence is recommended, however, and in
drowning victims the A-B-C approach is preferred because of the hypoxic nature of the cardiac arrest.
Emergency medical services (EMS) should be activated when the victim is found flaccid and unresponsive.
In certain situations CPR may be performed for up to 2 minutes before calling 911, but a 10-minute
interval is excessive. The Heimlich maneuver and attempts to positionally drain the airway may be harmful
and delay effective CPR.
The reported incidence of cervical spine injury in drowning victims is 0.009%. Attempts at cervical spine
immobilization are not necessary and may impede airway maintenance (SOR C).

44

Ref:

Berg RA, Hemphill R, Abella BS, et al: Part 5: Adult basic life support: 2010 American Heart Association Guidelines for
Cardiopulmonary Resuscitation and Emergency Cardiovascular Care. Circulation 2010;122(18 Suppl 3):S685-S705.
Vanden Hoek TL, Morrison LJ, Shuster M, et al: Part 12: Cardiac arrest in special situations: 2010 American Heart
Association Guidelines for Cardiopulmonary Resuscitation and Emergency Cardiovascular Care. Circulation 2010;122(18
Suppl 3):S829-S861.

Item 132
ANSWER:

Because of increased resistance to fluoroquinolones, ciprofloxacin is no longer recommended for the


treatment of gonorrhea. In addition, there appears to be emerging resistance to cephalosporins, as
evidenced by an increase in the minimum inhibitory concentrations of cephalosporins between 2000 and
2010. Unfortunately, no other well-studied and effective alternative antibiotic treatment regimens are
currently available. It also appears that gonococcal resistance to cefixime might develop before resistance
to ceftriaxone. As a result, in 2011 the Centers for Disease Control and Prevention recommended dual
treatment with ceftriaxone, 250 mg intramuscularly, and azithromycin, 1 g orally, as the most effective
treatment for uncomplicated gonorrhea.
Ref:

Centers for Disease Control and Prevention (CDC): Cephalosporin susceptibility among Neisseria gonorrhoeae
isolatesUnited States, 20002010. MMWR Morb Mortal Wkly Rep 2011;60(26):873-877.

Item 133
ANSWER:

Smoking cessation slows the decline of lung function in COPD. Long-acting $2-agonists, anticholinergic
agents, and inhaled corticosteroids are useful for improving the symptoms of COPD. They improve
exercise tolerance and quality of life, and can reduce the frequency of exacerbations. However, they do
not slow the progression of COPD. Oral corticosteroids, along with antibiotics, are useful in treating acute
exacerbations of COPD, but long-term treatment is not recommended.
Ref:

Drugs for chronic obstructive pulmonary disease. Treat Guidel Med Lett 2010;8(99):83-88.

Item 134
ANSWER:

This patient most likely has a psychogenic tremor, given its abrupt onset, spontaneous remission, changing
tremor characteristics, and extinction with distraction (SOR C). Other characteristics of this case that
suggest psychogenic tremor are the associated stressful life event, the patients employment in a health care
setting, and no evidence of disease by laboratory or radiologic investigations. In addition, the tremor
increases with attention and has been unresponsive to anti-tremor medications (SOR C).
Dystonic tremor is a rare tremor found in less than 1% of the population, and other signs of dystonia, such
as abnormal flexion of the wrists, are usually present. Essential tremor is an action tremor and is usually
postural; however, persons with essential tremor typically have no other neurologic findings. Essential
tremor typically improves with alcohol consumption (2 drinks/day). A cerebellar tremor is usually
associated with other neurologic signs, such as dysmetria (overshoot on finger-to-nose testing), dyssynergia
(abnormal heel-to-shin testing and/or ataxia), and hypotonia. A parkinsonian tremor is most often a resting
tremor, and although it may become less prominent with voluntary movement, it usually does not
spontaneously remit.
45

Ref:

Gupta A, Lang AE: Psychogenic movement disorders. Curr Opin Neurol 2009;22(4):430-436. 2) Crawford P, Zimmerman
EE: Differentiation and diagnosis of tremor. Am Fam Physician 2011;83(6):697-702.

Item 135
ANSWER:

Testosterone replacement therapy can improve many of the effects of hypogonadism. Beneficial effects
include improvements in mood, energy level, sexual functioning, sense of well-being, lean body mass and
muscle strength, erythropoiesis, bone mineral density, and cognition. However, there are also some risks
associated with testosterone use, including an increased risk for prostate cancer, worsening of symptoms
of benign prostatic hyperplasia, liver toxicity and tumor, worsening of sleep apnea and heart failure,
gynecomastia, infertility, and skin diseases. Testosterone replacement therapy is not appropriate in men
who are interested in maintaining fertility, as exogenous testosterone will suppress the
hypothalamic-pituitary-thyroid axis.
Ref:

Bassil N, Morley JE: Late-life onset hypogonadism: A review. Clin Geriatr Med 2010;26(2):197-222.

Item 136
ANSWER:

In addition to their lipid-lowering effects, statins have been shown to have plaque-stabilizing and vascular
anti-inflammatory effects. There is strong clinical evidence that perioperative statin therapy, even when
initiated within days of the procedure and without regard to lipid levels, significantly reduces
cardiovascular risk for patients undergoing vascular surgery (SOR A).
There is strong clinical evidence of benefit in perioperative cardiovascular risk reduction for continuation
of $-blockers before, during, and after vascular surgery in patients who have been on them for at least 4
weeks preoperatively (SOR A). However, in patients who have not been on a $-blocker for at least 14
weeks preoperatively, initiation prior to surgery may be harmful (SOR B).
Traditionally, aspirin has been discontinued prior to surgery for fear of increased surgical bleeding
complications. However, studies have shown that in most cases it is safe to continue low-dose aspirin in
the perioperative period, and doing so reduces cardiovascular complications. This is especially true for
patients with a past history of myocardial infarction or with coronary stents (SOR B).
Preoperative cardiac stress testing is of little value in patients with low or medium cardiovascular risk
status, such as the patient described here. Enoxaparin would not be indicated preoperatively in this patient.
Ref:

Holt NF: Perioperative cardiac risk reduction. Am Fam Physician 2012;85(3):239-246.

46

Item 137
ANSWER:

At every office visit, family physicians should encourage smokers to quit (SOR A). Patients who are ready
to quit may be helped by various pharmacologic treatments. Sustained-release bupropion and nicotine
replacement, especially gum and lozenges, may delay the weight gain often associated with smoking
cessation. Varenicline has a variety of side effects, including an increased risk for cardiovascular events
and a multitude of neuropsychiatric symptoms. It is also the agent most commonly associated with
postsmoking cessation weight gain. Clonidine, considered a second-line, off-label alternative for smoking
cessation, is not associated with weight gain.
Ref:

Larzelere MM, Williams DE: Promoting smoking cessation. Am Fam Physician 2012;85(6):591-598.

Item 138
ANSWER:

For the initial treatment of polymyalgia rheumatica, current evidence suggests using prednisone, 15 mg
daily, or its equivalent, with slow tapering. Relapses are more common with an initial dosage of 10 mg
daily, and slow tapering is associated with fewer relapses. Few patients require a dosage greater than 15
mg/day, which increases the risk for adverse effects.
Ref:

Hernndez-Rodriguez J, Cid MC, Lpez-Soto A, et al: Treatment of polymyalgia rheumatica: A systematic review. Arch
Intern Med 2009;169(20):1839-1850. 2) Moon KT: What is the best treatment for polymyalgia rheumatica? Am Fam
Physician 2010;81(6):788.

Item 139
ANSWER:

This patient has a high probability of pulmonary embolism, given his clinical presentation and recent
hospitalization with bed rest. Multidetector CT is the best initial test to confirm pulmonary embolism in
this situation. D-dimer testing is of limited value in patients with a high probability of pulmonary
embolism. If positive for deep-vein thrombosis, venous ultrasonography of the lower limbs can eliminate
the need for CT or lung scans, but this occurs in only about 10% of patients. Pulmonary angiography is
currently reserved for the rare case in which catheter-based treatment is indicated.
Ref:

Agnelli G, Becattini C: Acute pulmonary embolism. N Engl J Med 2010;363(3):266-274.

Item 140
ANSWER:

Paroxetine has been shown to cause the highest rate of sexual dysfunction among the SSRIs and other
antidepressants. The fewest sexual side effects occur with bupropion.
Ref:

Gartlehner G, Hansen RA, Morgan LC, et al: Comparative benefits and harms of second-generation antidepressants for
treating major depressive disorder: An updated meta-analysis. Ann Intern Med 2011;155(11):772-785.

47

Item 141
ANSWER:

Contrast-induced nephropathy is a concern in patients undergoing contrast studies, and can lead to
decreased renal function. Theoretically, this can cause an increased risk of lactic acidosis in patients taking
metformin. Current guidelines recommend stopping metformin use before imaging procedures that use
contrast, and restarting it 48 hours after the procedure if renal function is unchanged. The other drugs
listed do not carry this risk, although they can cause other problems in hospitalized patients, such as
hypoglycemia, depending on the situation.
Ref:

Sawin G, Shaughnessy AF: Glucose control in hospitalized patients. Am Fam Physician 2010;81(9):1121-1124.

Item 142
ANSWER:

When gynecomastia persists for a prolonged period, the initial glandular hyperplasia is transformed to a
progressive fibrosis and hyalinization. Surgery remains the mainstay of therapy. Medical management is
most useful when the onset is recent or to prevent the initial development of the problem. All the drugs
listed have been tried with varying success in this context, but their clinical usefulness is not established.
Ref:

Bembo SA, Carlson HE: Gynecomastia: Its features, and when and how to treat it. Cleve Clin J Med 2004;71(6):511-517.
2) Melmed S, Polonsky KS, Larsen PR, Kronenberg HM (eds): Williams Textbook of Endocrinology, ed 12. Elsevier
Saunders, 2011, pp 719-721.

Item 143
ANSWER:

This patients presentation is characteristic of vestibular neuronitis, a common condition affecting the
vestibular apparatus. The exact location and cause of the derangement is uncertain, although a viral or
post-viral cause has been postulated.
Benign positional vertigo is characterized by brief attacks of vertigo. Menieres disease is associated with
tinnitus and hearing loss. Migraines have a more gradual onset, and the symptoms of eustachian tube
dysfunction would be milder.
Ref:

Marx JA (ed): Rosens Emergency Medicine: Concepts and Clinical Practice, ed 7. Mosby Elsevier, 2010, pp 93-100.

Item 144
ANSWER:

Many patients are concerned about the risks associated with estrogen replacement therapy, and alternative
options should be addressed. Escitalopram has been shown to be effective for hot flashes in
postmenopausal women. This would be a reasonable choice for this patient, who also likely has depression.
Ref:

Freeman EW, Guthrie KA, Caan B, et al: Efficacy of escitalopram for hot flashes in healthy menopausal women: A
randomized controlled trial. JAMA 2011;305(3):267-274.

48

Item 145
ANSWER:

The range of normal hemoglobin values for healthy individuals varies with age, sex, pregnancy, smoking,
altitude, and ethnicity to an extent that an adjustment derived from population-based studies is appropriate
in each of these situations. A healthy individual should have their lifetime highest hemoglobin
concentrations at full-term birth, exclusive of any later changes from altitude or smoking. Hemoglobin
levels fall during the first 2 months of life and thereafter gradually increase until stabilizing at
approximately 6 months of age.
Studies of ethnic groups in the U.S. demonstrate no significant differences in normal hemoglobin values
among East Asians, Hispanics, Japanese, Native Americans, and non-Hispanic whites; hemoglobin values
of African-Americans tend to be 1 g/dL lower compared to the other U.S. ethnic groups studied. Living
at higher altitudes has a direct effect on hemoglobin levels, and a hemoglobin reference range adjustment
of +1 g/dL at 1000 meters up to +5.5 g/dL at 5000 meters is appropriate. Similarly, smoking increases
hemoglobin levels by 3 g/dL for a 1 pack/day smoker to as high as +7 g/dL for individuals smoking more
than 2 packs/day. The plasma volume expansion that occurs during pregnancy results in a 1.01.5 g/dL
reduction in normal hemoglobin levels.
Ref:

Nestel P: Adjusting Hemoglobin Values in Program Surveys. The International Nutritional Anemia Consultative Group,
2002. 2) Jopling J, Henry E, Wiedmeier SE, Christensen RD: Reference ranges for hematocrit and blood hemoglobin
concentration during the neonatal period: Data from a multihospital health care system. Pediatrics 2009;123(2):e333-e337.

Item 146
ANSWER:

Sudden cardiac death affects 500,000 people in the United States each year, causing more deaths than lung
cancer, breast cancer, and stroke combined. The most common final pathway is ventricular tachycardia
degenerating into ventricular fibrillation. The best predictor of sudden cardiac death is an ejection fraction
35%. Thus, it is critical for family physicians to evaluate the ejection fraction of patients with heart
disease.
Ref:

Turakhia MP: Sudden cardiac death and implantable cardioverter-defibrillators. Am Fam Physician 2010;82(11):1357-1366.

Item 147
ANSWER:

Clostridium difficile infection is a common cause of diarrhea in hospitalized patients, and recent antibiotic
use is a risk factor for infection. The bacteria can be spread in a hospital setting by contact, and contact
precautions with gown and gloves are indicated in addition to hand washing with soap and water to ensure
removal of spores (SOR A). Hand sanitizer is inadequate, as it does not kill the spores. Respiratory
precautions are not necessary.
Ref:

Cohen SH, Gerding DN, Johnson S, et al: Clinical practice guidelines for Clostridium difficile infection in adults: 2010
update by the Society for Healthcare Epidemiology of America (SHEA) and the Infectious Diseases Society of America
(IDSA). Infect Control Hosp Epidemiol 2010;31(5):431-455.

49

Item 148
ANSWER:

It is estimated that about one-third of patients with bipolar disorder seek medical care within a year of the
onset of symptoms, but that nearly 70% do not receive an accurate diagnosis. The symptoms can often be
subtle and may be attributed to other causes by patients or their loved ones.
A diagnosis of attention-deficit disorder requires that a patients symptoms be present since early
childhood, although they are sometimes not recognized at the time. This patient and his girlfriend have
both acknowledged that he is not his usual self. He presents with increased self-esteem, a decreased need
for sleep, pressured/tangential speech, and irritability, which point to the possibility of a manic or
hypomanic episode. Together these symptoms suggest bipolar disease (SOR C). Patients with full-blown
mania are often out of touch with reality and easy to identify. However, patients with hypomania consider
themselves to have increased well-being and productivity, and will not always seek attention or consider
themselves to have a problem.
Other symptoms that should alert the physician to this diagnosis include substance abuse (present in over
70% of cases) and involvement in other pleasurable but destructive activities such as overspending or
hypersexuality. If substance abuse is present, however, it must be addressed before making a diagnosis
of bipolar disorder. Bipolar disorder is highly genetic, and asking about affected first degree family
members can often assist in making the diagnosis.
Ref:

Loganathan M, Lohano K, Roberts RJ, et al: When to suspect bipolar disorder. J Fam Pract 2010;59(12):682-688.

Item 149
ANSWER:

This patient has asymptomatic hyperthyroidism, which is more common in the elderly. Elevated T4 and
markedly suppressed TSH are diagnostic. Common causes include Graves disease, toxic adenoma,
multinodular goiter, thyroiditis, and use of iodine-containing medications such as amiodarone.
Amiodarone-associated hyperthyroidism may be related to either iodine excess or a toxic effect on the
gland, causing thyroiditis (level of evidence 3). Lithium is associated with hypothyroidism.
Ref:

Donangelo I, Braunstein GD: Update on subclinical hyperthyroidism. Am Fam Physician 2011;83(8):933-938.

Item 150
ANSWER:

Allergic reactions to medications have four primary mechanisms, referred to as Gell and Coombs
classifications. The most frequent forms are type I reactions, which are immediate and mediated through
IgE, and type IV reactions, which are delayed and mediated through T-cell hypersensitization. Severe type
I reactions are often referred to as anaphylaxis and are the most likely to be life threatening with very little
warning. Recognition of the early signs of anaphylaxis is the first step in preventing such catastrophes.

50

Anaphylactic reactions result from a massive release of histamine and start with pruritus around the mouth,
on the scalp, and on the palms and soles; flushing of the face and neck, with rhinitis and conjunctivitis;
angioedema of the oral mucosa, especially of the pharynx and larynx; severe urticaria; dyspnea and
bronchospasm (especially in known asthmatics); and hypotension. A delay in lifesaving therapy during this
phase will result in full shock, hypotension, and death. Type IV reactions usually result in benign, diffuse
erythematous macules on the trunk and proximal extremities, often referred to as a drug rash. These
reactions infrequently become more severe and rarely are life threatening. In severe cases the lesions
become painful and palpable, and may involve blistering, mucositis, and ecchymosis.
Ref:

Schnyder B: Approach to the patient with drug allergy. Med Clin North Am 2010;94(4):665-679. 2) Scherer K, Bircher
AJ: Danger signs in drug hypersensitivity. Med Clin North Am 2010;94(4):681-689.

Item 151
ANSWER:

Vaginal atrophy is a common symptom accompanying menopause. Local application of estrogen is the
most effective treatment (SOR A) and is FDA approved for this indication. Efficacy ranges from 80% to
100%. All formulations are equally effective, so patient preference should drive the choice. Potential
adverse effects of vaginal estrogen include candidal infections, bleeding, burning with application, and
breast pain.
It is not necessary to add an oral progestogen to the local estrogen treatment. Vaginal estrogen does not
lead to endometrial proliferation, so endometrial protection in a patient who still has her uterus is not
necessary.
Ref:

Hill DA, Hill SR: Counseling patients about hormone therapy and alternatives for menopausal symptoms. Am Fam
Physician 2010;82(7):801-807. 2) North American Menopause Society: Estrogen and progestogen use in postmenopausal
women: 2010 position statement of The North American Menopause Society. Menopause 2010;17(2):242-255.

Item 152
ANSWER:

The frequency of head lice infestations has increased in recent years, and resistance to permethrin is now
common. Permethrin is unlikely to be effective in this child since her siblings infestations have failed to
respond to it. Of the other choices, only benzyl alcohol lotion is approved for use in children under 2 years
of age.
Ref:

Spinosad (Natroba) topical suspension for head lice. Med Lett Drugs Ther 2011;53:50-51. 2) Kliegman RM, Stanton BF,
Geme JW III, et al (eds): Nelson Textbook of Pediatrics, ed 19. Elsevier Saunders, 2011, pp 2321-2322.

51

Item 153
ANSWER:

Premenstrual syndrome (PMS) may be diagnosed when recurrent psychological and physical symptoms
occur only during the week prior to menses. The presence of more severe affective and somatic symptoms
that cause significant dysfunction in a patients social and work life is more consistent with premenstrual
dysphoric disorder. Both pyridoxine (vitamin B6), 50100 mg/day, and chasteberry, 20 mg/day, have been
shown in randomized, controlled trials to reduce the symptoms of PMS compared with placebo. No good
evidence supports the use of vitamin E, saffron, St. Johns wort, or soy.
Ref:

Biggs WS, Demuth RH: Premenstrual syndrome and premenstrual dysphoric disorder. Am Fam Physician
2011;84(8):918-924.

Item 154
ANSWER:

The U.S. Preventive Services Task Force and the American Heart Association/American Stroke
Association recommend not performing carotid artery screening with ultrasonography or other screening
tests in patients without neurologic symptoms because the harms outweigh the benefits. In the general
population, screening tests for carotid artery stenosis would result in more false-positive results than
true-positive results. This would lead to surgical procedures that are not indicated or to confirmatory
angiography. As a result of these procedures, some patients would suffer serious harms such as death,
stroke, or myocardial infarction, which outweigh the potential benefit surgical treatment may have in
preventing stroke.
Ref:

US Preventive Services Task Force: Screening for carotid artery stenosis: Recommendation statement. Am Fam Physician
2008;77(7):1006-1010.

Item 155
ANSWER:

This patient has rheumatoid arthritis (RA) by symptoms and physical findings. A positive latex fixation
test for rheumatoid factor is not necessary for the diagnosis. A negative rheumatoid factor does not
exclude RA, and a positive rheumatoid factor is not specific. Rheumatoid factor is found in the serum of
approximately 85% of adult patients with RA; in subjects without RA, the incidence of positive rheumatoid
factor is 1%5% and increases with age.
The ANA test is positive in at least 95% of patients with systemic lupus erythematosus, but in only about
35% of patients with RA. Elevation of the erythrocyte sedimentation rate is seen in many patients with
RA, and the degree of elevation roughly parallels disease activity. At a mean of 6 months after the onset
of Lyme disease, 60% of patients in the United States have brief attacks of asymmetric, oligoarticular
arthritis, primarily in the large joints and especially in the knee.
Ref:

Klippel JH, Stone JH, Crofford LJ, et al (eds): Primer on the Rheumatic Diseases, ed 13. Springer, 2008, pp 114-121.

52

Item 156
ANSWER:

This patient has a disequilibrium type of dizziness. Causes of this include medication side effects,
Parkinsons disease, and peripheral neuropathy. In this patient the history and examination do not indicate
a specific cause. She is on several medications, and one or more could be contributing to her symptoms.
A trial of medication reduction should be considered before ordering additional studies.
Ref:

Post RE, Dickerson LM: Dizziness: A diagnostic approach. Am Fam Physician 2010;82(4):361-368, 369.

Item 157
ANSWER:

Among the causes of food poisoning, Staphylococcus aureus is associated with the shortest incubation
period (16 hours). Most cases are related to contamination of food by infected human carriers. Leaving
food to cool slowly at room temperature allows organisms that produce enterotoxins to multiply. Common
food sources include ham, poultry, potato or egg salad, cream, and pastries.
Campylobacter jejuni infections can be subclinical or symptomatic. Symptoms usually occur within 24
days of exposure to the organism in food or water. A prodrome of fever, headache, and myalgias occurs
2448 hours before the diarrheal symptoms begin. Vomiting is usually not a symptom of the infection.
Clostridium botulinum is usually associated with canned foods. Enterohemorrhagic Escherichia coli
produces a Shiga-like toxin, which kills intestinal epithelial cells. While the symptoms are like those of
food poisoning, they are much more severe and are usually associated with bloody diarrhea (SOR C).
Ref:

DuPont HL: Bacterial diarrhea. N Engl J Med 2009;361(16):1560-1569. 2) Longo DL, Fauci AS, Kasper DL, et al (eds):
Harrisons Principles of Internal Medicine, ed 18. McGraw-Hill, 2012, p 1088.

Item 158
ANSWER:

A single intra-articular injection has little or no effect on glycemic control (SOR A). Soft-tissue or
peritendinous injections can affect blood glucose levels for 521 days, however, and diabetic patients
should closely monitor blood glucose levels for 2 weeks following these injections.
Ref:

Stephens MB, Beutler AI, OConnor FG: Musculoskeletal injections: A review of the evidence. Am Fam Physician
2008;78(8):971-976.

53

Item 159
ANSWER:

Constitutional delay of growth and puberty (CDGP) tends to be inherited. Bone age is delayed, but growth
potential is often normal. LH and FSH are elevated in hypergonadotropic hypogonadism, but this is not
characteristic of CDGP. Thyrotropin is most often elevated in hypothyroidism, which can cause a
secondary delay in growth and puberty. Anosmia is characteristic of Kallmann syndrome, but not CDGP.
Puberty is also delayed in this form of hypogonadotropic hypogonadism. Prolactin is elevated in some
pituitary tumors and by dopamine-blocking agents (SOR C).
Ref:

Palmert MR, Dunkel L: Delayed puberty. N Engl J Med 2012;366(5):443-453.

Item 160
ANSWER:

The combination of elevated LDH and elevated unconjugated bilirubin with otherwise normal liver enzyme
levels suggests hemolysis. Gilberts syndrome would not explain the LDH elevation. Hepatitis is unlikely
with normal transaminase levels.
Ref:

Tintinalli JE, Kelen GD, Stapczynski JS (eds): Emergency Medicine: A Comprehensive Study Guide, ed 7. McGraw-Hill,
2011, pp 571-572. 2) Longo DL, Fauci AS, Kasper DL, et al (eds): Harrisons Principles of Internal Medicine, ed 18.
McGraw-Hill, 2012, pp 2527-2531.

Item 161
ANSWER:

Routine screening of the general population for ovarian cancer is not recommended by any professional
society.
Ref:

Clarke-Pearson DL: Screening for ovarian cancer. N Engl J Med 2009;361(2):170-177.

Item 162
ANSWER:

The preferred anticoagulant for venous thrombosis during pregnancy is low molecular weight heparin.
Unfractionated heparin requires more monitoring and may increase the risk of heparin-induced
thrombocytopenia. Warfarin should not be used during pregnancy but may be used in women who are
breastfeeding (SOR B).
Ref:

Dresang LT, Fontaine P, Leeman L, King VJ: Venous thromboembolism during pregnancy. Am Fam Physician
2008;77(12):1709-1716. 2) Longo DL, Fauci AS, Kasper DL, et al (eds): Harrisons Principles of Internal Medicine, ed
18. McGraw-Hill, 2012, pp 57-58.

54

Item 163
ANSWER:

Eruptive xanthomas like these are associated with elevated triglycerides, obesity, alcohol abuse, diabetes
mellitus, and estrogen or retinoid therapies. The lesions of molluscum contagiosum can be distinguished
from these xanthomas by the characteristic central umbilication of molluscum.
Ref:

Holsinger JM, Campbell SM, Witman P: Multiple erythematous-yellow, dome-shaped papules. Am Fam Physician
2010;82(5):517.

Item 164
ANSWER:

There is no specific therapy for anemia of chronic disease except to manage or treat the underlying
disorder. Iron therapy is of no benefit, but erythropoietin may be helpful in some patients. There is no
available data to suggest that combination therapy or prednisone is beneficial for this disorder.
Ref:

Bross MH, Soch K, Smith-Knuppel T: Anemia in older persons. Am Fam Physician 2010;82(5):480-487. 2) Goldman L,
Schafer AI (eds): Goldmans Cecil Medicine, ed 24. Elsevier Saunders, 2011, pp 1042-1043.

Item 165
ANSWER:

Enuresis alarms should be offered as initial treatment for bed-wetting, based on randomized, controlled
trials and cost-effectiveness evidence. Desmopressin can also be considered if the child or parents do not
want to try an alarm. Restriction of fluids during the day should not be recommended, as it is important
that children have enough to drink. Waking a child and carrying him or her to the toilet has not been
shown to have a long-term effect on bed-wetting. Oxybutynin and imipramine should only be considered
in cases where bed-wetting does not respond to initial treatment.
Ref:

Nunes VD, OFlynn N, Evans J, et al: Management of bedwetting in children and young people: Summary of NICE
guidance. BMJ 2010;341:c5399.

Item 166
ANSWER:

The U.S. Medical Eligibility Criteria for Contraceptive Use were created to guide health care providers
in assessing the safety of contraceptive use for patients with specific conditions. Category 1 includes
conditions for which no restrictions exist for use of the contraceptive method. Category 2 indicates that
the method generally can be used, but careful follow-up may be required. Category 3 is used to classify
conditions for which the method usually is not recommended unless more-preferred methods are not
available or acceptable. Category 4 comprises conditions that represent an unacceptable health risk if the
method is used. For combined hormonal contraceptives, migraine headaches with aura at any age are
classified as category 4 because of the increased risk of ischemic stroke. A family history of breast cancer
is category 1, rheumatoid arthritis treated by immunosuppression is category 2, a BMI 30 kg/m2 is
category 2, and ovarian cancer is category 1.

55

Ref:

Centers for Disease Control and Prevention (CDC): US Medical Eligibility Criteria for Contraceptive Use, 2010. MMWR
Recomm Rep 2010;59(RR-4):1-86.

Item 167
ANSWER:

Acute bronchiolitis is a viral illness most frequently caused by the respiratory syncytial virus. Its peak
incidence occurs at approximately 6 months of age. The illness frequently causes a few days of mild upper
respiratory symptoms, followed by increased coughing and wheezing. Examination often reveals tachypnea
and use of the accessory muscles of respiration, such as intercostal retractions. Acute asthma is uncommon
in the first year of life, and is difficult to diagnose without recurrent episodes or prior respiratory
problems. Croup usually presents with stridor, and pertussis and pneumonia do not usually present with
wheezing.
Ref:

Kliegman RM, Stanton BF, Geme JW III, et al (eds): Nelson Textbook of Pediatrics, ed 19. Elsevier Saunders, 2011, pp
1456-1459.

Item 168
ANSWER:

All antipsychotic agents can prolong ventricular repolarization, leading to a prolonged QT interval, which
can in turn lead to torsades de pointes and sudden cardiac death. Although all antipsychotics can affect
EKG intervals, the agents with the greatest propensity to prolong QTc are thioridazine, pimozide,
droperidol, and ziprasidone. The incidence of sudden cardiac death among patients taking antipsychotics
is about twice that of the general population.
Ref:

Muench J, Hamer AM: Adverse effects of antipsychotic medications. Am Fam Physician 2010;81(5):617-622.

Item 169
ANSWER:

Stress fractures are caused by repetitive loading that exceeds the bones ability to heal. They occur more
commonly in female athletes. Patients should be evaluated for risk factors such as eating disorders,
menstrual irregularities, and chronic medical conditions. Lower-extremity alignment, gait, and strength
should also be evaluated. High-risk fractures such as those of the femoral neck, anterior cortex of the tibia,
or proximal fifth metatarsal should be referred to an orthopedist, as there is a high likelihood of
fracture-related complications.
Ref:

Sanderlin BW, Raspa RF: Common stress fractures. Am Fam Physician 2003;68(8):1527-1532. 2) Joy EA, Van Hala S,
Cooper L: Health-related concerns of the female athlete: A lifespan approach. Am Fam Physician 2009;79(6):489-495.

Item 170
ANSWER:

Cardioselective $-blockers, such as metoprolol, should not be withheld from patients with COPD.
Metoprolol could be started at a low dosage in this patient. Nadolol, timolol, and sotalol are not
cardioselective.

56

Ref:

Salpeter S, Ormiston T, Salpeter E: Cardioselective beta-blockers for chronic obstructive pulmonary disease. Cochrane
Database Syst Rev 2005;(4):CD003566. 2) Navas EV, Taylor DO: Q: Can patients with COPD or asthma take a
beta-blocker? Cleve Clin J Med 2010;77(8):498-499.

Item 171
ANSWER:

While most patients with heart failure should be treated with an ACE inhibitor and a diuretic, a subset of
patients with heart failure present with only fatigue or mild dyspnea on exertion, and no evidence of
volume overload. Since ACE inhibitors alone appear to prevent or slow the development of heart failure
in patients with asymptomatic left ventricular dysfunction, it is reasonable to start an ACE inhibitor such
as lisinopril in patients with very mild symptoms and observe to see if the symptoms resolve. The other
medications listed are useful in the treatment of heart failure but would not be appropriate as initial
treatment in the patient described.
Ref:

Hunt SA, Abraham WT, Chin MH, et al: 2009 focused update incorporated into the ACC/AHA 2005 Guidelines for the
Diagnosis and Management of Heart Failure in Adults: A report of the American College of Cardiology
Foundation/American Heart Association Task Force on Practice Guidelines: Developed in collaboration with the
International Society for Heart and Lung Transplantation. Circulation 2009;119(14):e391-479. 2) Bonow RO, Mann DL,
Zipes DP, Libby P (eds): Braunwalds Heart Disease: A Textbook of Cardiovascular Medicine, ed 9. Elsevier Saunders,
2011, pp 549-564.

Item 172
ANSWER:

Pneumococcal 13-valent vaccine produces a satisfactory immune response in 1-year-old children, while
polyvalent vaccine does not cause a good antibody response in children under the age of 2 years. Neither
vaccine is available orally, and cost is not a factor. The 13-valent vaccine requires multiple doses. The
vaccine cannot be combined with MMR in a single injection, but can be administered concurrently with
routine childhood immunizations at a separate site using a separate syringe.
Ref:

23-Valent pneumococcal polysaccharide vaccine: WHO position paper. Wkly Epidemiol Rec 2008;83(42):373-384. 2)
Nuorti JP, Whitney CG; Centers for Disease Control and Prevention (CDC): Prevention of pneumococcal disease among
infants and childrenUse of 13-valent pneumococcal conjugate vaccine and 23-valent pneumococcal polysaccharide vaccine:
Recommendations of the Advisory Committee on Immunization Practices (ACIP). MMWR Recomm Rep 2010;59(RR11):1-18.

Item 173
ANSWER:

This patient most likely has Charcot foot (neuropathic arthropathy). This is an uncommon condition, most
often found in patients with at least a 10-year history of type 1 or 2 diabetes. Patients have peripheral
neuropathy and typically present with painless swelling of a foot. About 50% of patients can recall minor
trauma preceding the onset of symptoms. Early radiologic findings may be normal, but as the condition
progresses plain films may reveal bony fractures, fragmentation, and bone destruction. On examination
the foot is either stiff or hypermobile, warm, erythematous, and edematous. Crepitus can be felt in later
stages of the disease. Laboratory results can be normal or show increases in the WBC count. Any patient
with long-standing diabetes mellitus and a warm, red, swollen foot should be presumed to have Charcot
foot and have the foot immobilized immediately to protect it from stress and to prevent further destruction
and disability (SOR C).
57

The differential diagnosis of Charcot foot includes osteomyelitis, which can be difficult to distinguish.
Patients with osteomyelitis can have skin ulcers that may be probed to the bone. They will often be febrile,
with higher erythrocyte sedimentation rates and WBC counts than patients with Charcot foot. Aspiration
of the joint fluid is helpful in distinguishing between the two conditions.
Aspiration can also help distinguish between acute gout and Charcot foot. Patients may have pain and
stiffness in their foot with either condition. Patients with gout may have an elevated uric acid level, and
plain films may reveal tophi.
Patients with osteoarthritis are unlikely to have warm, erythematous skin overlying the involved joint.
Symptoms often have an insidious onset rather than the more acute onset of Charcot foot.
Ref:

Sommer TC, Lee TH: Charcot foot: The diagnostic dilemma. Am Fam Physician 2001;64(9):1591-1598. 2) Kronenberg
HM, Melmed S, Polonsky K, et al: Williams Textbook of Endocrinology, ed 11. Saunders, 2008, pp 1482-1483.

Item 174
ANSWER:

General recommendations regarding prevention of recurrent nephrolithiasis include increasing fluid intake
up to 2 L of water daily (SOR B); greater volumes may lead to electrolyte disturbances and are not
recommended. More specific dietary recommendations depend on the stone type. If the stone is not
recovered, the type may be inferred from a 24-hour urine collection for calcium, phosphorus, magnesium,
uric acid, and oxalate.
Approximately 60% of all stones in adults are calcium oxalate. Uric acid stones account for up to 17% of
stones and, like cystine stones, form in acidic urine. Alkalinization of the urine to a pH of 6.57.0 may
reduce stone formation in patients with these types of stones. This includes a diet with plenty of fruits and
vegetables, and limiting acid-producing foods such as meat, grains, dairy products, and legumes. Drinking
mineral water, which is relatively alkaline with a pH of 7.07.5, is also recommended. Restriction of
dietary oxalates has not been shown to be effective in reducing stone formation in most patients.
Acidification of the urine to a pH 7.0 is recommended for patients with the less common calcium
phosphate and struvite stones. This can be accomplished by consumption of at least 16 oz of cranberry
juice per day, or by taking betaine, 650 mg three times daily.
Ref:

Frassetto L, Kohlstadt I: Treatment and prevention of kidney stones: An update. Am Fam Physician 2011;84(11):1234-1242.

Item 175
ANSWER:

The statement, It is time to consider withdrawal of care, can make patients think that the physician no
longer wants to care for them. It would be better to ask, Do you think it is time for us to consider a
different type of treatment that focuses on your symptoms? Telling the individual that you want to provide
intense coordinated care with a team that will treat symptoms and maintain comfort is a way of involving
hospice without making the patient feel hopeless.
Ref:

Ngo-Metzger Q, August KJ, Srinivasan M, et al: End-of-life care: Guidelines for patient-centered communication. Am Fam
Physician 2008;77(2):167-174.

58

Item 176
ANSWER:

De Quervains tenosynovitis is a tenosynovitis of the first dorsal compartment of the wrist, specifically a
chronic inflammation of the extensor pollicis brevis and abductor pollicis longus tendons. It is a fairly
common cause of pain in the distal forearm. The diagnosis is made by physical examination. Pain,
tenderness, and occasionally swelling are present on the radial side of the wrist. The pain is exacerbated
by passive wrist ulnar deviation while the thumb is flexed and the fingers curled around it.
These symptoms are not typical for rheumatoid disease, and cervical radiculopathy would not cause radial
tenderness. Carpal joint arthritis would be more distal. The sesamoid bones are common and of no clinical
significance. Carpal tunnel syndrome is caused by an entrapment neuropathy of the median nerve as it
traverses the carpal tunnel. Symptoms typically include pain and paresthesias in the hand. Numbness
occurs in the first two fingers, in the distribution of the median nerve. Thenar muscle weakness is a later
sign.
Ref:

Frontera WR, Silver JK, Rizzo TD Jr (eds): Essentials of Physical Medicine and Rehabilitation, ed 2. Saunders Elsevier,
2008, pp 129-131.

Item 177
ANSWER:

This patients examination is consistent with female pattern hair loss. Women with female pattern hair loss
who also have a history of abnormal menses, infertility, cystic acne, and hirsutism should have an
evaluation for hyperandrogenism. Minoxidil 2% topically is the only treatment approved by the FDA for
treating female pattern hair loss in women over 18, but a hyperandrogenic state may limit the response to
minoxidil. If the hyperandrogenism evaluation is normal, spironolactone, 100200 mg daily, may slow the
rate of hair loss. Approximately 90% of such women report a modest decrease in hair loss with this
treatment.
Ref:

Mounsey AL, Reed SW: Diagnosing and treating hair loss. Am Fam Physician 2009;80(4):356-362.

Item 178
ANSWER:

Tourettes syndrome is often associated with psychiatric comorbidities, mainly


attention-deficit/hyperactivity disorder and obsessive-compulsive disorder. The other conditions listed are
not associated with Tourettes syndrome.
Ref:

Kenney C, Kuo SH, Jimenez-Shahed J: Tourettes syndrome. Am Fam Physician 2008;77(5):651-658.

Item 179
ANSWER:

Plantar fasciitis affects more than 1 million people in the United States each year. Risk factors include
excessive pronation, running, obesity, and prolonged standing. Patients often have pain when they get out
of bed and take their first steps in the morning, or after prolonged sitting. Palpation usually causes pain
in the medial plantar calcaneal region. The pain is described as sharp and stabbing.
59

A sudden onset of ecchymosis and heel pain is more consistent with a diagnosis of plantar fascia rupture.
Pain in the region of the posterior medial ankle is more consistent with posterior tibial tendinitis. Burning
pain in the medial plantar region is more consistent with medial calcaneal and abductor digiti quinti nerve
entrapment.
Ref:

Goff JD, Crawford R: Diagnosis and treatment of plantar fasciitis. Am Fam Physician 2011;84(6):676-682.

Item 180
ANSWER:

Seborrheic dermatitis is one of the more common skin conditions affecting infants within the first few
months of life. The characteristic reddish, waxy rash most commonly involves the scalp (cradle cap), but
can also appear on the face, ears, neck, skin folds, and diaper area. While the rash is similar to that of
atopic dermatitis, seborrheic dermatitis is not associated with pruritus. Although the rash can appear
alarming to parents, reassurance that the condition can be expected to resolve within a few months is the
most appropriate management.
Ref:

OConnor NR, McLaughlin MR, Ham P: Newborn skin: Part I. Common rashes. Am Fam Physician 2008;77(1):47-52.

Item 181
ANSWER:

Somatization disorder usually begins in the teens or twenties and is characterized by multiple unexplained
physical symptoms, insistence on surgical procedures, and an imprecise or inaccurate medical history.
Abuse of alcohol, narcotics, or other drugs is also common in these patients.
Hypochondriacs are overly concerned with bodily functions, and can often provide accurate, extensive,
and detailed medical histories. Malingering is an intentional pretense of illness to obtain personal gain.
Patients with panic disorder have episodes of intense, short-lived attacks of cardiovascular, neurologic,
or gastrointestinal symptoms. Generalized anxiety disorder is characterized by unrealistic worry about life
circumstances accompanied by symptoms of motor tension, autonomic hyperactivity, or vigilance and
scanning.
Ref:

American Psychiatric Association: Diagnostic and Statistical Manual of Mental Disorders, ed 4. American Psychiatric
Association, 1994, pp 448-450. 2) Sadock BJ, Sadock VA, Ruiz P (eds): Kaplan & Sadocks Comprehensive Textbook of
Psychiatry, ed 9. Lippincott Williams & Wilkins, 2009, pp 1927-1935.

Item 182
ANSWER:

This patient has mild acute croup, also known as laryngotracheobronchitis. She has no signs of pneumonia,
epiglottitis, or bacterial tracheitis. Acute croup is almost always viral in nature, with parainfluenza viruses
being the most common etiologic agents. A single dose of oral dexamethasone has been shown to benefit
children with even mild croup, presumably by decreasing edema of the laryngeal mucosa. Inhaled
epinephrine is helpful in severe croup with signs of respiratory distress. Inhaled albuterol is used to treat
asthma. Oseltamivir would be appropriate treatment for influenza, and azithromycin for bacterial
pneumonia.
Ref:

Zoorob R, Sidani M, Murray J: Croup: An overview. Am Fam Physician 2011;83(9):1067-1073.

60

Item 183
ANSWER:

Chronic radiation proctitis develops months to years after radiation exposure and is characterized by pain
with defecation, along with diarrhea and sometimes rectal bleeding. On colonoscopy, the mucosa is pale
and friable with telangiectases which are sometimes large, multiple, and serpiginous.
Ref:

Berkey FJ: Managing the adverse effects of radiation therapy. Am Fam Physician 2010;82(4):381-388, 394.

Item 184
ANSWER:

It is recommended that asthmatics, once stabilized, be taken off long-acting $-agonists and maintained on
an inhaled corticosteroid such as fluticasone. It is not recommended to change from a combination inhaled
corticosteroid/long-acting $-agonist to a long-acting anticholinergic agent. Montelukast can be used for
maintenance, but inhaled corticosteroids are preferable.
Ref:

Ducharme F, Schwartz Z, Hicks G, Kakuma R: Addition of anti-leukotriene agents to inhaled corticosteroids for chronic
asthma. Cochrane Database Syst Rev 2004;(2):CD003133. 2) Chowdhury BA, Dal Pan G: The FDA and safe use of
long-acting beta-agonists in the treatment of asthma. N Engl J Med 2010;362(13):1169-1171.

Item 185
ANSWER:

This patient almost certainly has sarcoidosis. The diagnosis is supported by a compatible clinical and
radiographic presentation, and histologic evidence of noncaseating granulomas on a biopsy (without
organisms or particles). In patients who present with Lfgren syndrome (erythema nodosum, hilar
adenopathy, and polyarthralgias), a probable diagnosis of sarcoidosis can be made without a biopsy. In
all other cases a biopsy should be performed on the most accessible organ, such as the skin or peripheral
lymph nodes.
While the thorax is the most common site of disease, skin involvement occurs in at least 30% of patients
and is often missed. This patient was diagnosed with psoriasis, which may have been another manifestation
of sarcoidosis. Cutaneous sarcoidosis presents as single foci or crops, and is often attributed to other
causes, perhaps because of its highly variable manifestations such as macular-papular, nodular,
psoriatic-like, and hypomelanotic lesions. Careful skin examination is warranted because biopsy of a
sarcoidal lesion has a high diagnostic yield.
Ref:

Iannuzzi MC, Fontana JR: Sarcoidosis: Clinical presentation, immunopathogenesis, and therapeutics. JAMA
2011;305(4):391-399.

61

Item 186
ANSWER:

Problems with gait and balance increase in frequency with advancing age and are the result of a variety
of individual or combined disease processes. Findings may be subtle initially, making it difficult to make
an accurate diagnosis, and knowing the relative frequencies of primary causes may be useful for
management. A cautious gait (broadened base, slight forward leaning of the trunk, and reduced arm swing)
may be the first manifestation of many diseases, or it may just be somewhat physiologic if not excessive.
In the past, a problematic gait abnormality in an elderly person was generally termed a senile gait if there
was no clear diagnosis; it is more accurate, however, to describe this as an undifferentiated gait problem
secondary to subclinical disease. From the long list of potential causes, arthritic joint disease is by far the
most likely to be seen in the family physicians office, accounting for more than 40% of total cases. It most
frequently causes an antalgic gait characterized by a reduced range of motion. The patient favors affected
joints by limping or taking short, slow steps.
Ref:

Salzman B: Gait and balance disorders in older adults. Am Fam Physician 2010;82(1):61-68. 2) Lam R: Office management
of gait disorders in the elderly. Can Fam Physician 2011;57(7):765-770.

Item 187
ANSWER:

Pregnancy with an intrauterine device in place is rare but does occur. Removal of an in situ intrauterine
device in early pregnancy reduces the risks of spontaneous abortion, preterm labor, and sepsis, so gentle
removal should be accomplished as soon as the pregnancy becomes known.
Ref:

Cunningham FG, Leveno KJ, Bloom SL, et al (eds): Williams Obstetrics, ed 23. McGraw-Hill Medical, 2010, p 686.

Item 188
ANSWER:

This patient meets the criteria for antisocial personality disorder, including age over 18, evidence of
conduct disorder in childhood, a pattern of irresponsible and antisocial behavior since age 15, and absence
of schizophrenia or manic episodes. Although the patient has some features of borderline personality
disorder, such as unstable relationships, the persistently aggressive nature and lack of remorse are much
more typical of antisocial personality. While the boasting quality of the patient might appear somewhat
grandiose, there are no other features to suggest mania. Abused child reaction formation is not a
recognized diagnosis in the Diagnostic and Statistical Manual of Mental Disorders. Schizotypal personality
disorder is not usually associated with such pervasive antisocial behavior and violence.
Ref:

American Psychiatric Association: Diagnostic and Statistical Manual of Mental Disorders, ed 4. American Psychiatric
Association, 1994, pp 645-650. 2) Sadock BJ, Sadock VA, Ruiz P (eds): Kaplan & Sadocks Comprehensive Textbook of
Psychiatry, ed 9. Lippincott Williams & Wilkins, 2009, pp 2220-2221.

62

Item 189
ANSWER:

Although Medicare does not pay for an annual physical, it does provide for annual preventive screening
services, including a complete health history and an array of screening measures for depression, fall risk,
cognitive problems, and other challenges. The physical examination conducted as part of the annual
wellness visit includes measurement of blood pressure and weight, a vision check, and hearing evaluation,
as well as additional elements depending on the individuals health risks.
While questioning the patient or caregiver regarding perceived hearing difficulties may suffice when
screening for hearing loss, screening for vision loss requires use of a standard screening tool.
Documentation of visual acuity by use of the Snellen chart is an accepted means of screening for visual
acuity in the primary care setting (SOR A). Vision screening will not pick up age-related macular
degeneration or cataracts, however.
Ref:

Screening for visual acuity in older adults: Recommendation statement. US Preventive Services Task Force, 2009. 2)
Hughes C: Medicare annual wellness visits made easier. Fam Pract Manag 2011;18(4):10-14.

Item 190
ANSWER:

Depot medroxyprogesterone acetate and the combination contraceptive vaginal ring are the most effective
hormonal contraceptives for obese women because they do not appear to be affected by body weight.
Women using the combination contraceptive patch who weight 90 kg may experience decreased
contraceptive efficacy. Obese women using oral contraceptives may also have an increased risk of
pregnancy.
Ref:

Gordon L, Thakur N, Atlas M, Januchowski R: What hormonal contraception is most effective for obese women? J Fam
Pract 2007;56(6):471-473.

Item 191
ANSWER:

There are several possible causes of secondary hypertension in young adults age 1939, including
coarctation of the aorta, thyroid dysfunction, renal parenchymal disease, and fibromuscular dysplasia.
Fibromuscular dysplasia is more common in females, and has a predilection for causing stenosis of the
renal arteries. The diagnosis can be made using MRI with gadolinium contrast media, or with CT
angiography.
Middle-aged adults (age 4064) are more likely to have primary aldosteronism (evaluated with an
aldosterone/renin ratio), sleep apnea, pheochromocytoma (associated with elevated metanephrines), or
Cushings syndrome (elevated 24-hour urinary cortisol). The patient described has no signs or symptoms
of any of these problems.
Ref:

Viera AJ, Neutze DM: Diagnosis of secondary hypertension: An age-based approach. Am Fam Physician
2010;82(12):1471-1478.

63

Item 192
ANSWER:

The female athlete triad is a relatively common condition in athletes, and is characterized by amenorrhea,
disordered eating, and osteoporosis. It is more common in sports that promote lean body mass. Female
athletes should be screened for the disorder during their preparticipation evaluations. Individuals who
present with one or more components of the triad should be evaluated for the other components. This
patient evidences disordered eating (low BMI for age) and secondary amenorrhea, and should be screened
for osteoporosis using a DXA scan. The International Society for Clinical Densitometry recommends using
the Z-score, rather than the T-score, when screening children or premenopausal women. The T-score is
based on a comparison to a young adult at peak bone density, whereas the Z-score uses a comparison to
persons of the same age as the patient. A Z-score less than 2.0 indicates osteoporosis. The American
College of Sports Medicine defines low bone density as a Z score of 1.0 to 2.0.
An EKG is not required in this patient since she has normal vital signs. Pelvic ultrasonography is not
necessary unless an abnormal finding is identified on a pelvic examination. Abdominopelvic CT would be
inappropriate given the patients age and lack of abdominopelvic symptoms such as pain or a mass. A
nuclear bone scan likewise is not recommended, as it is not used to diagnose osteoporosis (SOR C).
Ref:

Joy EA, Van Hala S, Cooper L: Health-related concerns of the female athlete: A lifespan approach. Am Fam Physician
2009;79(6):489-495. 2) Mendelsohn FA, Warren MP: Anorexia, bulimia, and the female athlete triad: Evaluation and
management. Endocrinol Metab Clin North Am 2010;39(1):155-167.

Item 193
ANSWER:

To address and minimize perioperative neurologic morbidity in patients undergoing coronary artery bypass
grafting (CABG), individualized surgical management strategies are now recommended. These address
patient risk factors for postoperative stroke, such as carotid stenosis, hypertension, older age, a past history
of stroke, small-vessel disease in the brain, and diabetes mellitus. Because concomitant carotid disease is
often associated with CAD, pre-CABG carotid Doppler ultrasonography is routinely recommended.
Several approaches have been evaluated for decreasing the risk associated with carotid stenosis.
Performing both carotid and CABG surgery at the same time increases stroke risk, and no studies have
compared doing one before the other. While the use of statins has increased in patients with systemic
atherosclerosis, the roles of both postoperative statins and $-blockers are still controversial. The only
treatment that has been shown to reduce postoperative cerebrovascular events is the use of aspirin in the
first 48 hours after surgery.
Ref:

Selnes O, Gottesman RF, Grega MA, et al: Cognitive and neurologic outcomes after coronary-artery bypass surgery. N
Engl J Med 2012;366(3):250-257.

Item 194
ANSWER:

Based on a review of randomized, controlled trials, screening mammography reduces breast cancer
mortality for women between 40 and 49 years of age (SOR A). The absolute risk reduction is less for
women in this age group than for older women, however. For technical reasons, and because of greater
breast density, the false-positive and false-negative rates may be higher in this group than in older women.
Radiation risk is also greater in younger women.
64

Ref:

Qaseem A, Snow V, Sherif K, et al: Screening mammography for women 40 to 49 years of age: A clinical practice
guideline from the American College of Physicians. Ann Intern Med 2007;146(7):511-515. 2) Krishnaiah PB, Nunes NL,
Safranek S: FPINs clinical inquiries: Screening mammography for reducing breast cancer mortality. Am Fam Physician
2012;85(2):176-183.

Item 195
ANSWER:

Pneumococcal vaccine is usually given only once to individuals 65 years of age. A repeat dose may be
given 5 years later for those at higher risk. Immunization is also recommended for younger persons with
chronic medical problems, such as heart disease, diabetes mellitus, renal failure, and sickle cell anemia,
as well as those who have undergone splenectomy or who work or live with high-risk persons.
Ref:

Centers for Disease Control and Prevention (CDC); Advisory Committee on Immunization Practices: Updated
recommendations for prevention of invasive pneumococcal disease among adults using the 23-valent pneumococcal
polysaccharide vaccine (PPSV23). MMWR Morb Mortal Wkly Rep 2010;59(34):1102-1106.

Item 196
ANSWER:

All of the diagnoses listed may cause elevations of serum calcium, but malignancy, sarcoidosis, Paget's
disease, and hypervitaminosis D are all associated with suppressed levels of parathyroid hormone. While
the parathyroid levels in this patient are within the normal range, they are inappropriately high for the level
of serum calcium and suggest hyperparathyroidism, the most common cause of hypercalcemia in this age
group.
Ref:

Carroll MF, Schade DS: A practical approach to hypercalcemia. Am Fam Physician 2003;67(9):1959-1966. 2) Longo DL,
Fauci AS, Kasper DL, et al (eds): Harrisons Principles of Internal Medicine, ed 18. McGraw-Hill, 2012, pp 3096-3111.

Item 197
ANSWER:

Research has consistently confirmed that stimulant medications are the most efficacious first-line treatment
for children with attention-deficit/hyperactivity disorder (ADHD) (SOR A). No research supports the
notion that the use of a stimulant in ADHD patients will promote addiction. To the contrary, some evidence
suggests that ADHD patients who take stimulant medication have lower rates of drug abuse than those who
do not. Diversion and misuse of prescription stimulants is a growing concern, however, and the use of a
long-acting stimulant can decrease the chances for diversion.
There are a number of well-supported behavioral interventions for ADHD. Most behavioral approaches
focus on rewarding desired behavior and applying consequences for unwanted behavior to gradually
reshape the childs thinking and actions. Interventions that help reinforce parental involvement include
support groups, which connect parents who have children with similar problems, and parenting skills
training, which gives parents techniques and tools for managing their childs behavior. Psychotherapy and
cognitive-behavioral therapy have little or no documented effectiveness for the treatment of ADHD.

65

A multicenter, randomized study comparing the effectiveness of multimodal treatment (combined


behavioral interventions and pharmacotherapy) with either treatment alone showed that combination
treatment and pharmacotherapy alone yielded similar results and each was more effective than behavioral
treatment alone or standard care in reducing core ADHD symptoms. A tool kit has been developed by the
American Academy of Pediatrics and the National Initiative for Childrens Healthcare Quality to help
physicians improve the management of ADHD. While the second edition is only available in print form,
the first edition can be downloaded free at http://www.nichq.org/adhd_tools.html.
Ref:

Rader R, McCauley L, Callen EC: Current strategies in the diagnosis and treatment of childhood
attention-deficit/hyperactivity disorder. Am Fam Physician 2009;79(8):657-665. 2) Antshel KM, Hargrave TM, Simonescu
M, et al: Advances in understanding and treating ADHD. BMC Med 2011;9:72.

Item 198
ANSWER:

The low TSH level suggests hyperthyroidism. If TSH is <0.5 :U/mL, the immediate next step is to
measure free T3 and free T4 levels (SOR C), which are elevated in hyperthyroidism and normal in
subclinical hyperthyroidism. If levels of free T3 and free T4 are elevated and the underlying cause of
hyperthyroidism is unknown, then it is advisable to order a 24-hour radioactive iodine (RAI) uptake test.
With Graves disease, RAI uptake is increased and diffuse, whereas with toxic multinodular goiter it is
increased and nodular (SOR A). If RAI uptake is low, subacute thyroiditis should be suspected and could
be confirmed by measuring levels of thyroglobulin (SOR A). Levothyroxine is not indicated, as it is used
to treat hypothyroidism. Fasting does not significantly affect TSH levels.
Ref:

Baskin HJ, Cobin RH, Duick DS, et al: American Association of Clinical Endocrinologists medical guidelines for clinical
practice for the evaluation and treatment of hyperthyroidism and hypothyroidism. Endocr Pract 2002;8(6):457-469. 2)
Boelaert K, Torlinska B, Holder RL, Franklyn JA: Older subjects with hyperthyroidism present with a paucity of symptoms
and signs: A large cross-sectional study. J Clin Endocrinol Metab 2010;95(6):2715-2726. 3) Ghandour A, Reust C:
Hyperthyroidism: A stepwise approach to management. J Fam Pract 2011;60(7):388-395.

Item 199
ANSWER:

The patient has symptoms consistent with chronic bacterial prostatitis: irritative voiding symptoms;
testicular, perineal, and low back pain; recurrent urinary tract infections; and distal penile pain. His
symptoms have also been present for more than 3 months. Because chronic bacterial prostatitis is a
bacterial infection, an appropriate antibiotic with good tissue penetration in the prostate should be selected.
Fluoroquinolones have the best tissue concentration and are recommended as first-line agents.
Penicillin derivatives, commonly used to treat acute prostatitis, have not been shown to provide good
symptom relief for chronic bacterial prostatitis. "-Blockers are second-line agents for treating chronic
pelvic pain. Transrectal ultrasonography is indicated in patients whose acute prostatitis fails to resolve and
who have a persistent fever or whose maximal temperature is not trending downward after 36 hours. In
this case a prostatic abscess should be suspected, and transrectal ultrasonography can facilitate the
diagnosis. Prostate-specific antigen is a screening test for prostatic malignancy and would contribute
nothing to the management of this patients problem.
Ref:

Sharp VJ, Takacs EB, Powell CR: Prostatitis: Diagnosis and treatment. Am Fam Physician 2010;82(4):397-406.

66

Item 200
ANSWER:

Observational studies have consistently shown that alcohol use has a J-shaped curve for several health
effects. Small amounts of alcohol on a daily basis (less than 1 drink/day for women, and 12 drinks/day
for men) are associated with an 18% lower risk for all-cause mortality and a 30% decreased risk for
coronary heart disease. As the use of alcohol increases these benefits disappear and even reverse, showing
a dose-dependent increase in all-cause mortality when women consume more than 2 drinks/day or men
over 4 drinks/day. At this level of alcohol use, rates increase for hypertension, cancer, stroke, heart
failure, dementia, and diabetes mellitus.
Ref:

OKeefe JH, Bybee KA, Lavie CJ: Alcohol and cardiovascular health: The razor-sharp double-edged sword. J Am Coll
Cardiol 2007;50(11):1009-1014. 2) OKeefe JH, Carter MD, Lavie CJ: Primary and secondary prevention of
cardiovascular diseases: A practical evidence-based approach. Mayo Clin Proc 2009;84(8):741-757. 3) Ronksley PE, Brien
SE, Turner BJ, et al: Association of alcohol consumption with selected cardiovascular disease outcomes: A systematic
review and meta-analysis. BMJ 2011;342:d671.

Item 201
ANSWER:

The only FDA-approved oral medication for the treatment of interstitial cystitis is pentosan polysulfate
sodium, which is thought to repair the urothelium (SOR B). Trimethoprim/sulfamethoxazole and
nitrofurantoin are indicated for urinary tract infections (UTIs), but usually not in cases of cystitis with no
infection. In addition, this patient has already received empiric treatment for a UTI despite having multiple
negative urine cultures. Ibuprofen is an anti-inflammatory medication commonly used to treat pain but is
not specifically indicated for interstitial cystitis. While tricyclic antidepressants such as amitriptyline have
been used to treat interstitial cystitis, fluoxetine is not generally recommended.
Ref:

Davis EL, El Khoudary SR, Talbott EO, et al: Safety and efficacy of the use of intravesical and oral pentosan polysulfate
sodium for interstitial cystitis: A randomized double-blind clinical trial. J Urol 2008;179(1):177185. 2) French LM,
Bhambore N: Interstitial cystitis/painful bladder syndrome. Am Fam Physician 2011;83(10):1175-1181.

Item 202
ANSWER:

A previous history of depression is the strongest risk factor for depression during pregnancy. Other risk
factors include childhood abuse, smoking, age under 20, and low socioeconomic status, especially without
social support. A family history of hyperthyroidism is not a risk factor.
Ref:

Stewart DE: Depression during pregnancy. N Engl J Med 2011;365(17):1605-1611.

67

Item 203
ANSWER:

This patient has perlche, or angular cheilitis. Most cases are secondary to moisture from patients licking
their lips, promoting a monilial or staphylococcal infection. Other causes include contact and irritant
dermatitis. Underlying HIV infection, celiac disease, or vitamin B12 and iron deficiencies have also been
reported. Treatment may include appropriate topical creams such as mupirocin or antifungal agents, or
low-potency nonfluorinated corticosteroid creams for irritant or contact causes.
Ref:

Gonsalves WC, Chi AC, Neville BW: Common oral lesions: Part I. Superficial mucosal lesions. Am Fam Physician
2007;75(4):501-507.

Item 204
ANSWER:

Almost all patients with ocular problems should have visual acuity testing before anything else is done
(level of evidence 3, SOR A). If this is difficult, a local anesthetic may be applied. The main exception
to this rule is a chemical burn of the eye, which should be irrigated for 30 minutes before further
evaluation or treatment is undertaken..
Ref:

Pokhrel PK, Loftus SA: Ocular emergencies. Am Fam Physician 2007;76(6):829-836.

Item 205
ANSWER:

Systemic corticosteroids are recognized for their dramatic impact on both the subjective and objective
course of poison ivy dermatitis. Oral prednisone at an initial dosage of 1 mg/kg/day tapered over 1421
days is the standard regimen. Complications can result from the use of shorter prepackaged courses of
corticosteroid therapy, resulting in significant rebound flares. These products usually begin with an initial
dosage approximately half that of the recommended dosage, with the course tapering too rapidly.
Over-the-counter topical hydrocortisone is ineffective for all but the mildest cases, and once the disease
is established, superpotent topical corticosteroids do little to alter the overall course and natural history.
Antihistamines and compresses provide some symptomatic relief, but do little to alter the course of
established disease.
Ref:

Habif TP: Clinical Dermatology: A Color Guide to Diagnosis and Therapy, ed 5. Mosby Elsevier, 2010, pp 138-139.

68

Item 206
ANSWER:

The U.S. Preventive Services Task Force recommends that all adults be screened for colon cancer
beginning at age 50 and continue regular screening until age 75 (SOR A). They recommend against
continued routine screening in previously screened adults 7585 years of age and against any screening
in adults over 85 (SOR A). Most organizations do not recommend a particular screening method, but
instead list screening options, including fecal occult blood testing, flexible sigmoidoscopy, and
colonoscopy. The recommended interval for fecal occult blood testing is every year. There is new evidence
based on randomized, controlled trials that participation and detection rates for advanced adenomas and
cancer are higher for immunochemical fecal testing than for stool guaiac testing (SOR A). As long as
results are normal, screening colonoscopy is recommended at 10-year intervals and screening
sigmoidoscopy at 5-year intervals.
Ref:

Hewitson P, Glasziou P, Irwig L, et al: Screening for colorectal cancer using the faecal occult blood test, Hemoccult.
Cochrane Database Syst Rev 2007;(1):CD001216. 2) US Preventive Services Task Force: Screening for colorectal cancer:
US Preventive Services Task Force recommendation statement. Ann Intern Med 2008;149(9):627-637. 3) Wilkins T,
Reynolds PL: Colorectal cancer: A summary of the evidence for screening and prevention. Am Fam Physician
2008;78(12):1385-1392, 1393-1394. 4) van Rossum LG, van Rijn AF, Laheij RJ, et al: Random comparison of guaiac and
immunochemical fecal occult blood tests for colorectal cancer in a screening population. Gastroenterology
2008;135(1):82-90.

Item 207
ANSWER:

Teenage mortality is an important public health issue because the majority of deaths among teenagers are
caused by external causes of injury such as accidents, homicide, and suicide. The leading causes of death
for the teenage population remained constant throughout the period 19992006: accidents (48% of deaths),
homicide (13%), suicide (11%), cancer (6%), and heart disease (3%). Motor vehicle accidents accounted
for 73% of all deaths from unintentional injury.
Ref:

Minino AM: Mortality among teenagers aged 1219 years: United States 19992006. National Center for Health Statistics,
NCHS data brief no 37, 2010.

Item 208
ANSWER:

Interventions recommended for hospitalized older patients to reduce the risk of hospital-induced disability
include minimizing restricted diets. Bed rest orders should be avoided, with recommendations that the
patient ambulate 34 times/day and be out of bed and in a chair for all meals. This patient is disoriented
and probably has delirium. Restraints should be avoided if possible, and should be limited if they become
necessary. This would also apply to functional restraints, such as indwelling urinary catheters, IV poles,
nasal cannulas, continuous pulse oximetry, and telemetry, which all increase the risk of delirium.
Ref:

Covinsky KE, Pierluissi E, Johnston CB: Hospitalization-associated disability: She was probably able to ambulate, but
Im not sure. JAMA 2011;306(16):1782-1793.

69

Item 209
ANSWER:

The first step in diagnosing deep-vein thrombosis (DVT) is to complete a validated clinical prediction
inquiry such as the Wells Clinical Prediction Rule in order to estimate the pretest probability of DVT. The
Wells criteria include such factors as active cancer, calf swelling, pitting edema, prolonged inactivity, or
major surgery within the previous 12 weeks.
The next step for patients with a low pretest probability of DVT is a high-sensitivity D-dimer assay, with
a negative result indicating a low likelihood of DVT (SOR A). D-dimer is a degradation product of
cross-linked fibrin blood clots and is usually elevated in patients with DVT, although it can also be elevated
with other conditions such as recent surgery, hemorrhage, trauma, pregnancy, or cancer. If the assay is
negative, the likelihood of DVT is very small.
In this case, the patient has several factors listed in the Wells criteria, indicating a high pretest probability
of DVT. D-dimer testing would not be useful, as the next step in this patients evaluation should be
imaging. Ultrasonography is the best test for symptomatic proximal-vein thrombosis, with a sensitivity
ranging between 89% and 96% (SOR A). Although ultrasonography is the most appropriate first imaging
test, contrast venography is considered the definitive test to rule out the diagnosis of DVT if there is still
a high degree of suspicion after negative ultrasonography.
Helical CT is commonly used to detect pulmonary embolism but is not routinely recommended to diagnose
DVT. Similarly, MRI is not routinely recommended for detecting DVT.
Ref:

Segal JB, Eng J, Tamariz LJ, Bass EB: Review of the evidence on diagnosis of deep venous thrombosis and pulmonary
embolism. Ann Fam Med 2007;5(1): 63-73. 2) Qaseem A, Snow V, Barry P, et al: Current diagnosis of venous
thromboembolism in primary care: A clinical practice guideline from the American Academy of Family Physicians and the
American College of Physicians. Ann Fam Med 2007;5(1):57-62.

Item 210
ANSWER:

The extended flat pharmacokinetic curve of long-acting insulin analogues makes once-daily administration
of larger doses of insulin possible. Such treatment should, in theory, provide increased flexibility with
regard to the timing of injections and improve compliance. This should improve control of the patients
diabetes, reduce the risk of hypoglycemia, and improve overall patient satisfaction. To date, however, the
only proven benefit of treatment with insulin analogues is a reduction in the low rate of symptomatic,
nocturnal, and overt hypoglycemia experienced by patients treated with isophane insulin. Although the total
cost of treatment with insulin analogues is higher, a Cochrane review of the limited number of studies
comparing insulin treatments showed no statistically significant differences in the hemoglobin A1c levels
measured at the end of the studies in any treatment group (SOR C). Significant changes in morbidity,
mortality, or quality of life have not been demonstrated (SOR C).
Ref:

Schooff MD, Gupta L: Are long-acting insulin analogues better than isophane insulin? Am Fam Physician
2008;77(4):447-449.

70

Item 211
ANSWER:

In 2011, the American College of Physicians published new guidelines on COPD management. For
patients with COPD who are symptomatic and have an FEV1 less than 60% of predicted, the
recommendation is monotherapy with either a long-acting inhaled anticholinergic (tiotropium) or a
long-acting inhaled $-agonist such as salmeterol or formoterol. This is in addition to rescue therapy with
a short-acting inhaled bronchodilator such as albuterol. Long-acting inhaled anticholinergics and
long-acting inhaled $-agonists reduce exacerbations and improve quality of life. The evidence is
inconclusive with regard to their effect on mortality, hospitalizations, and dyspnea.
Inhaled corticosteroids have been found to be better than placebo for decreasing COPD exacerbations, but
their side-effect profile keeps them from being preferred as monotherapy. Neither inhaled mast-cell
stabilizers nor inhaled antihistamines are recommended as first-line agents for the treatment of COPD.
Ref:

Qaseem A, Wilt TJ, Weinberger SE, et al: Diagnosis and management of stable chronic obstructive pulmonary disease:
A clinical practice guideline update from the American College of Physicians, American College of Chest Physicians,
American Thoracic Society, and European Respiratory Society. Ann Intern Med 2011;155(3):179-191. 2) Armstrong C:
ACP updates guideline on diagnosis and management of stable COPD. Am Fam Physician 2012;85(2):204-205.

Item 212
ANSWER:

This patient has grade 2 internal hemorrhoids. These protrude with defecation but reduce spontaneously.
Sitz baths are commonly recommended, but a review of studies found no benefit from sitz baths for various
anorectal disorders, including hemorrhoids. A meta-analysis of seven randomized trials of patients with
symptomatic hemorrhoids showed that fiber supplementation with psyllium, sterculia, or unprocessed bran
decreased bleeding, pain, prolapse, and itching. No randomized, controlled trials support the use of
corticosteroid creams for treating hemorrhoidal disease. Topical diltiazem and topical lidocaine have been
shown to provide pain relief postoperatively following excision of external hemorrhoids.
Ref:

Mounsey AL, Halladay J, Sadiq TS: Hemorrhoids. Am Fam Physician 2011;84(2):204-210.

Item 213
ANSWER:

Sensitivity is defined as the percentage of patients with a disease who have a positive test for the disease
in question. Specificity is the percentage of patients without the disease who have a negative test. The
positive predictive value is the percentage of patients with a positive or abnormal test who have the disease
in question. The negative predictive value is the percentage of patients with a negative or normal test who
do not have the disease in question. Likelihood ratios correspond to the clinical impression of how well
a test rules in or rules out a given disease.
Ref:

Flaherty RJ: A simple method for evaluating the clinical literature. Fam Pract Manag 2004;11(5):47-52. 2) Rakel RE,
Rakel DP (eds): Textbook of Family Medicine, ed 8. Elsevier Saunders, 2011, pp 112-119.

71

Item 214
ANSWER:

Thrombotic thrombocytopenic purpura (TTP) is an emergent condition that can result in up to 30%
mortality. Prompt hospitalization with plasma exchange is the preferred treatment. Patients with TTP
present with nonspecific symptoms such as fever, abdominal pain, nausea, and weakness. Patients may
also exhibit neurologic deficits. Microangiopathic anemia is also likely to be present, as evidenced by
schistocytes on a peripheral smear and elevated levels of LDH and nucleated RBCs.
Congenital thrombocytopenia is a benign condition in which patients have long-standing low platelet counts
and/or a family history of thrombocytopenia. It is usually asymptomatic but a concomitant bleeding
diathesis may occur.
Gestational thrombocytopenia is also benign and asymptomatic. It is often confused with mild immune
thrombocytopenic purpura. Platelet counts rarely drop below 70,000/mm3. There is no associated fetal
thrombocytopenia. Preeclampsia and HELLP syndrome should also be ruled out. Platelet counts return
to normal after delivery (SOR C).
Drug-induced thrombocytopenia can be severe, but platelet counts do not usually drop below 20,000/mm3.
It is characterized by an abrupt drop in the platelet count within a week of starting the offending medication
and resolves within 2 weeks after the medication is stopped.
Lyme disease can be associated with a transient thrombocytopenia. Patients present with common
symptoms of Lyme disease, such as fever, myalgias, and rash. The thrombocytopenia resolves with
treatment of the underlying infection.
Ref:

Hoffman R, Benz EJ Jr, Shattil SJ, et al: Hematology: Basic Principles and Practice, ed 5. Churchill Livingstone Elsevier,
2009, pp 2100-2101. 2) Gauer RL, Braun MM: Thrombocytopenia. Am Fam Physician 2012;85(6):612-622.

Item 215
ANSWER:

Influenza should be diagnosed on the basis of clinical signs and symptoms rather than diagnostic testing.
Antiviral treatment is not recommended in otherwise healthy adults and children. Symptomatic treatment
should be initiated with over-the-counter antipyretics and anti-inflammatory medications, and aspirin should
be avoided due to the risk of Reyes syndrome. Antibiotics are indicated only when a bacterial coinfection
is diagnosed and not for prophylaxis.
Ref:

Erlikh IV, Abraham S, Kondamudi VK: Management of influenza. Am Fam Physician 2010;82(9):1087-1095.

72

Item 216
ANSWER:

This patient does not meet the LDL-cholesterol goals for a diabetic patient and therefore needs adjustment
of his antihyperlipidemic regimen. In June 2011, the Food and Drug Administration recommended limiting
the use of the highest dosage of simvastatin (80 mg/day) because of concerns about an increased risk of
muscle damage. This dosage should only be used in patients who have already been taking 80 mg/day for
12 months or more without evidence of muscle injury, and it should not be started in new patients. This
patient should be switched to an alternative medication that provides a greater reduction of
LDL-cholesterol, such as atorvastatin. Gemfibrozil is contraindicated for use with simvastatin because it
can raise simvastatin drug levels and increase the risk of myopathy.
Ref:

FDA drug safety communication: New restrictions, contraindications, and dose limitations for Zocor (simvastatin) to reduce
the risk of muscle injury. US Food and Drug Administration, 2011. 2) PL Detail-Document: New restrictions,
contraindications, and dose limitations for Zocor (simvastatin). Pharmacists Letter/Prescribers Letter. July 2011.

Item 217
ANSWER:

Polycystic ovary syndrome may be the most common cause of female infertility, affecting 6%8% of
women. Some patients with polycystic ovary syndrome have hyperandrogenism, elevated levels of
luteinizing hormone, and hyperinsulinemia. While early studies supported the use of metformin to increase
fertility, a more recent study has shown that only 7% of women treated with metformin were able to
conceive, whereas 22% of women treated with clomiphene citrate had a live birth. Spironolactone is useful
for treating hirsutism, but not infertility. Since levels of other hormones are already elevated, the other
measures listed would not be of benefit.
Ref:

Legro RS, Barnhart HX, Schlaff WD, et al: Clomiphene, metformin, or both for infertility in the polycystic ovary
syndrome. N Engl J Med 2007;356(6):551-566. 2) Melmed S, Polonsky KS, Larsen PR, Kronenberg HM (eds): Williams
Textbook of Endocrinology, ed 12. Elsevier Saunders, 2011, pp 631-632.

Item 218
ANSWER:

The American College of Rheumatology criteria for the diagnosis of systemic lupus erythematosus (SLE)
includes the presence of 4 of 11 criteria. One criterion is evidence of the presence of an immunologic
disorder. Evidence of an immunologic disorder includes a positive finding of antiphospholipid antibodies,
based upon one of the following: an abnormal serum level of immunoglobulin G or M anticardiolipin
antibodies, a positive lupus anticoagulant test, or a false-positive serologic test for syphilis. The other test
results listed are not criteria for the diagnosis of SLE.
Ref:

Klippel JH, Stone JH, Crofford LJ, et al (eds): Primer on the Rheumatic Diseases, ed 13. Springer, 2008, pp 303-314.
2) Gibson K, Goodemote P, Johnson S: Antibody testing for systemic lupus erythematosus. Am Fam Physician
2011;84(12):1407-1409.

73

Item 219
ANSWER:

This patient has stage 4 chronic kidney disease with a glomerular filtration rate (GFR) <30 mL/min. This
puts her at high risk for developing nephrogenic systemic fibrosis. The FDA recommends against using
gadolinium-based contrast agents in patients with acute or chronic kidney disease and a GFR <30 mL/min.
While anaphylaxis is possible, this patient would not be at a high risk for this. Dermatomyositis, focal
seizures, and hypertensive crisis are not associated with gadolinium-based contrast agents.
Ref:

Schlaudecker JD, Bernheisel CR: Gadolinium-associated nephrogenic systemic fibrosis. Am Fam Physician
2009;80(7):711-714.

Item 220
ANSWER:

Trimethoprim/sulfamethoxazole is the drug of choice for treating Pneumocystis jiroveci pneumonia.


Atovaquone has been shown to be very effective for treating mild to moderate Pneumocystis jiroveci
pneumonia and is also very well tolerated, and would be the first choice for a patient with a sulfa allergy.
Clindamycin plus primaquine is also effective therapy, but clindamycin is not effective as monotherapy.
Azithromycin and rifampin are not effective against this organism. Other treatment options include
pentamidine, dapsone plus trimethoprim, and clindamycin plus primaquine.
Ref:

Mandell GL, Bennett JE, Dolin R (eds): Mandell, Douglas, and Bennetts Principles and Practice of Infectious Diseases,
ed 7. Churchill Livingstone, 2009, pp 1858-1874.

Item 221
ANSWER:

Because of the high prevalence of reduced gastric acidity related to either endogenous causes or
medications such as proton pump inhibitors, calcium carbonate is best taken with meals to optimize
absorption. Calcium citrate, which is well absorbed regardless of gastric acidity, may be taken with or
without food. The other medications listed do not impair calcium absorption.
Ref:

Lewiecki EM: Osteoporosis. Ann Intern Med 2011;155(1):ITC1-1-ITC1-15.

74

Item 222
ANSWER:

Joint pain in the presence of fever with no apparent source indicates a possible infection, malignancy, or
rheumatologic condition and requires further workup. Laboratory evaluation, including a CBC, a
C-reactive protein level, and an erythrocyte sedimentation rate can help assess for these conditions, even
though none of the tests is sufficiently sensitive to rule out these diseases, and they are not specific to a
single disease entity. Knee joint aspiration would be indicated to rule out septic arthritis in the presence
of a joint effusion. If the hip were painful or had decreased range of motion, then ultrasonography could
help identify a hip joint effusion, which would need to be aspirated. MRI may be needed in this patient,
but it would likely require sedation and thus is more invasive. Starting with laboratory work is a good first
step toward identifying the source of his pain and fever.
Ref:

Sawyer JR, Kapoor M: The limping child: A systematic approach to diagnosis. Am Fam Physician 2009;79(3):215-224.

Item 223
ANSWER:

Extracorporeal shock wave therapy is effective for calcific tendinitis of the rotator cuff. Side effects include
bruising and pain. Needling and irrigation, physical therapy, and cortisone injections are sometimes used
in patients with acute symptoms. Endoscopic and open surgical treatments are alternatives to extracorporeal
shock wave therapy in refractory cases. Extracorporeal shock wave therapy does not have an established
role in gout, rotator cuff tear, frozen shoulder, or hooked acromion.
Ref:

Gerdesmeyer L, Wagenpfeil S, Haake M, et al: Extracorporeal shock wave therapy for the treatment of chronic calcifying
tendonitis of the rotator cuff: A randomized controlled trial. JAMA 2003;290(19):2573-2580. 2) Murphy R, Carr A:
Clinical Evidence Handbook. Shoulder pain. Am Fam Physician 2011;83(2):137-138.

Item 224
ANSWER:

This patient has acute bronchitis. The most appropriate management option is to provide reassurance that
symptoms will likely resolve on their own within 3 weeks. Approximately 90% of cases are caused by
viruses, and antibiotics do not significantly change the course of the condition. For this reason, and
because of concerns about antibiotic resistance and side effects from antibiotic use, antibiotics should not
be used routinely for the treatment of acute bronchitis (SOR B). Despite this, approximately two-thirds of
patients in the United States diagnosed with bronchitis are still treated with antibiotics. Corticosteroids and
$-agonists are not indicated in the absence of asthma or wheezing on examination. Expectorants have not
been shown to be effective in the treatment of bronchitis (SOR B).
Ref:

Braman SS: Chronic cough due to acute bronchitis: ACCP evidence-based clinical practice guidelines. Chest 2006;129(1
Suppl):95S-103S. 2) Albert RH: Diagnosis and treatment of acute bronchitis. Am Fam Physician 2010;82(11):1345-1350.

75

Item 225
ANSWER:

Recommendations for reducing ventilator-associated pneumonia (VAP) include elevation of the head to
an angle of at least 30, noninvasive mechanical ventilation rather than intubation when appropriate, oral
intubation when an endotracheal tube is necessary, orogastric rather than nasogastric tubes, minimization
of sedation, administration of a proton pump inhibitor when prophylaxis is indicated, changing ventilator
tubing every 7 days or when it becomes soiled, avoidance or elimination of endotracheal tube leaks, good
technique in removal of condensate, and excellent hand hygiene. One study favored waiting more than 5
days before initiating tube feedings, as this reduced the incidence of VAP, although further data is needed
to confirm this.
Ref:

Koenig SM, Truwit JD: Ventilator-associated pneumonia: Diagnosis, treatment, and prevention. Clin Microbiol Rev
2006;19(4):637-657.

Item 226
ANSWER:

There are no proven therapies to reduce the severity of borderline personality disorder (SOR A). The most
promising psychological therapy is dialectic behavioral therapy (DBT). DBT is a multi-faceted program
specifically designed to treat borderline personality disorder. The few, small studies of DBT found
improvement in many symptoms of borderline personality disorder, but long-term data is lacking. Another
promising therapy is psychoanalytic-oriented day hospital therapy. Again, study sizes have been small and
data cannot be extrapolated to the population as a whole.
Omega-3 fatty acids, second-generation antipsychotics, and mood stabilizers have been shown to be helpful
for some symptoms of borderline personality disorder but not for overall severity. Their benefits are based
on single-study results and side effects were not addressed in the studies. SSRIs are not recommended for
borderline personality disorder unless there is a concomitant mood disorder.
Ref:

Binks CA, Fenton M, McCarthy L, et al: Psychological therapies for people with borderline personality disorder. Cochrane
Database Syst Rev 2006;(1):CD005652. 2) Sadock BJ, Sadock VA, Ruiz P (eds): Kaplan & Sadocks Comprehensive
Textbook of Psychiatry, ed 9. Lippincott Williams & Wilkins, 2009, p 2885. 3) Stoffers J, Vllm BA, Rcker G, et al:
Pharmacological interventions for borderline personality disorder. Cochrane Database Syst Rev 2010;(6):CD005653.

Item 227
ANSWER:

The drug most often recommended as first-line therapy for hypertension in pregnancy is labetalol. Reports
of an association of metoprolol with fetal growth restriction have given rise to the recommendation to avoid
its use in pregnancy. Both ACE inhibitors and angiotensin-receptor blockers are contraindicated in
pregnancy because of the risk of birth defects and fetal or neonatal renal failure. Immediate-release
nifedipine is not recommended due to the risk of hypotension.
Ref:

Leeman L, Fontaine P: Hypertensive disorders of pregnancy. Am Fam Physician 2008;78(1):93-100. 2) Seely EW, Ecker
J: Chronic hypertension in pregnancy. N Engl J Med 2011;365(5):439-446.

76

Item 228
ANSWER:

Staphylococcus aureus is the most common cause of acute infectious endocarditis worldwide. Additionally,
the most common cause of tricuspid valve endocarditis is intravenous drug abuse, and Staphylococcus
aureus is the infecting organism in 80% of tricuspid valve infections. Streptococcus viridans is also a
frequent cause of infectious endocarditis, with Enterococcus, Pseudomonas, and Cardiobacterium being
less likely causes.
Ref:

Bonow RO, Carabello BA, Chatterjee K, et al: 2008 Focused update incorporated into the ACC/AHA 2006 guidelines for
the management of patients with valvular heart disease: A report of the American College of Cardiology/American Heart
Association Task Force on Practice Guidelines (Writing Committee to Revise the 1998 Guidelines for the Management of
Patients With Valvular Heart Disease): Endorsed by the Society of Cardiovascular Anesthesiologists, Society for
Cardiovascular Angiography and Interventions, and Society of Thoracic Surgeons. Circulation 2008;118(15):e523-e661.
2) Fuster V, Walsh RA, Harrington RA (eds): Hursts The Heart, ed 13. McGraw-Hill, 2011, chap 86.

Item 229
ANSWER:

A number of measures for managing spinal compression fractures have been evaluated. The evidence for
recommending kyphoplasty is weak, and the evidence for recommending against vertebroplasty is strong.
The data on bracing is inconclusive, as is the recommendation for bed rest. Calcitonin has been shown to
reduce the incidence of recurrent fractures and may be useful in the relief of pain.
Ref:

Esses SI, McGuire R, Jenkins J, et al: The treatment of symptomatic osteoporotic spinal compression fractures. J Am Acad
Orthop Surg 2011;19(3):176-182.

Item 230
ANSWER:

Heart disease is the leading cause of death in the U.S., and this holds true for both men and women.
Among men the only ethnicity for which heart disease is not the most common cause of death is
Asian/Pacific Islander.
Ref:

Heron M: Deaths: Leading causes for 2006. Natl Vital Stat Rep 2010;58(14):12.

Item 231
ANSWER:

A middle-aged woman with pruritus and elevated levels of alkaline phosphatase, (-glutamyltransferase
(GGT), and antimitochondrial antibody titers is likely to have primary biliary cirrhosis. Levels of
5'-nucleotidase or GGT are usually elevated in parallel with those of alkaline phosphatase in patients with
liver disease, but not in patients with bone disorders. Infiltrative liver disease, as seen with sarcoidosis,
drug-induced cholestasis, and choledocholithiasis, is not associated with elevated antimitochondrial
antibody levels. Patients with choledocholithiasis will also usually have dilated hepatic ducts on
ultrasonography. This patient should next have a liver biopsy to confirm her diagnosis.

77

Ref:

Pratt DS, Kaplan MM: Evaluation of abnormal liver-enzyme results in asymptomatic patients. N Engl J Med
2000;342(17):1266-1271. 2) Longo DL, Fauci AS, Kasper DL, et al (eds): Harrisons Principles of Internal Medicine,
ed 18. McGraw-Hill, 2012, p 2595.

Item 232
ANSWER:

According to the U.S. Preventive Services Task Force (USPSTF), there is good evidence that screening
for Chlamydia infection in women who are at increased risk can reduce the incidence of pelvic
inflammatory disease, while the harms are minimal. The evidence regarding screening for cervical cancer
with Papanicolaou testing or human papillomavirus (HPV) testing, however, shows that the harms
outweigh any possible benefits. Harms include overdiagnosis and overtreatment, including invasive
cervical procedures that can affect future pregnancy outcomes. In addition, there is adequate evidence that
screening women younger than 21 years of age (regardless of sexual history) does not reduce the incidence
of cervical cancer or mortality compared with beginning screening at age 21. The USPSTF concludes that
the evidence is insufficient to recommend for or against routine screening for lipid disorders or intimate
partner violence in women this age.
Ref:

Screening for family and intimate partner violence: Recommendation statement. US Preventive Services Task Force, 2004.
2) Screening for chlamydial infection: Recommendation statement. US Preventive Services Task Force, 2007. 3) Screening
for lipid disorders in adults: Recommendation statement. US Preventive Services Task Force, 2008. 4) Screening for
cervical cancer: US Preventive Services Task Force recommendation statement. US Preventive Services Task Force, 2012.

Item 233
ANSWER:

Interactions between the physician and patient involve the patients family and friends, as well as others
who may be part of the patients social support system. These interactions, as well as the physician-patient
relationship itself, are also strongly influenced by ethnic, cultural, and spiritual values and by beliefs about
illness and approaches to treatment and ongoing care. Involvement of family members in a patients care
is advantageous to good communication and helpful for both accurate diagnosis and appropriate treatment
(SOR C).
Patient confidentiality should be protected, and the mere presence of other people accompanying the patient
does not automatically constitute permission to discuss private health information with them. The physician
should determine not only who is present with a patient, but also each persons reason for being present
(SOR C). Speaking to relatives or friends without the patient present may be appropriate under certain
circumstances but would not be the best option in this scenario. Constructing a genogram can also be
helpful, but that would not be the first priority in this case.
Ref:

Omole FS, Sow CM, Fresh E, et al: Interacting with patients family members during the office visit. Am Fam Physician
2011;84(7):780-784.

78

Item 234
ANSWER:

Radiographs confirm the clinical diagnosis of small-bowel obstruction in most patients and more accurately
define the site of obstruction. Small-bowel obstruction typically occupies the more central portions of the
abdomen. Patients with mechanical small-bowel obstruction usually have minimal or no colonic gas. Films
taken in the upright or lateral decubitus position in patients with small-bowel obstruction usually show
multiple gas-filled levels, with the distended bowel resembling an inverted U.
Patients with small-bowel obstruction are likely to be depleted of fluids and electrolytes, and will require
intravenous fluids, electrolyte management, and surgical evaluation.
Ref:

Marx JA (ed): Rosens Emergency Medicine: Concepts and Clinical Practice, ed 7. Mosby Elsevier, 2010, pp 1184-1188.
2) Townsend CM Jr, Beauchamp RD, Evers BM, et al: Sabiston Textbook of Surgery: The Biological Basis of Modern
Surgical Practice, ed 19. Saunders, 2012, pp 1236-1243.

Item 235
ANSWER:

The salient feature of atrial fibrillation is the absence of P waves, along with normal QRS complexes that
are irregular in time (irregularly irregular) and sometimes vary in amplitude. Sinus tachycardia,
paroxysmal tachycardia, multifocal atrial tachycardia, and atrial flutter are all associated with P waves that
are constantly related to QRS complexes, although they may sometimes be abnormal and difficult to
discern.
Ref:

Bonow RO, Mann DL, Zipes DP, Libby P (eds): Braunwalds Heart Disease: A Textbook of Cardiovascular Medicine, ed
9. Elsevier Saunders, 2011, pp 825-844. 2) Longo DL, Fauci AS, Kasper DL, et al (eds): Harrisons Principles of Internal
Medicine, ed 18. McGraw-Hill, 2012, pp 1880-1890.

Item 236
ANSWER:

Scleritis is an inflammatory disorder affecting the sclera, often associated with a connective tissue disorder
such as rheumatoid arthritis, systemic lupus erythematosus, Wegeners granulomatosis, polyarteritis
nodosa, or relapsing polychondritis. In the anterior form of scleritis, inflammation results in local or
diffuse erythema and thickening of the sclera. Patients present with either diffuse or focal ocular erythema,
tenderness, and pain. When the inflammation is focal, a tender nodule may be present. The initial
treatment of scleritis is an oral NSAID to help reduce ocular inflammation. Topical or systemic
corticosteroids may be used when NSAIDs fail or are contraindicated (SOR B).
Ref:

Longo DL, Fauci AS, Kasper DL, et al (eds): Harrisons Principles of Internal Medicine, ed 18. McGraw-Hill, 2012, p
229.

79

Item 237
ANSWER:

The condition shown is representative of a simple anterior dislocation of the lunate. The semilunar shape
of the lunate bone is displaced anterior to the distal radial articular surface. Occasionally, a transnavicular
fracture may occur along with this injury and is termed a trans-scaphoid perilunate fracture-dislocation.
There is no evidence in the radiograph shown, however, of dislocations of the other areas mentioned.
Ref:

DeLee JC, Drez D Jr, Miller MD (eds): DeLee & Drez's Orthopaedic Sports Medicine: Principles and Practice, ed 3.
Elsevier Saunders, 2010, pp 1321-1325.

Item 238
ANSWER:

The findings in this patient are consistent with a lung abscess caused by anaerobic organisms, which is
usually related to aspiration. Most patients have a history of compromised consciousness, such as a seizure
disorder or drug and alcohol abuse, and many have dental or gingival disease.
Routine bacteriologic studies of expectorated sputum are hampered by mouth contamination, but are useful
for detecting mycobacteria and other potential etiologic agents. An open lung biopsy, immediate
bronchoscopy, and transtracheal aspiration are useful when the patient has not responded to initial therapy.
Since this is a typical clinical picture for anaerobic lung abscess, the treatment of choice would be large
doses of intravenous clindamycin.
Ref:

Longo DL, Fauci AS, Kasper DL, et al (eds): Harrisons Principles of Internal Medicine, ed 18. McGraw-Hill, 2012, p
2145. 2) Bope ET, Kellerman RD (eds): Conns Current Therapy 2012. Elsevier Saunders, 2012, 366-367.

Item 239
ANSWER:

Right ventricular infarction is most frequently an extension of an inferior myocardial infarction (MI).
Right-sided precordial leads may need to be evaluated to document this on the EKG. Hypotension during
the acute event, especially after nitrate administration, is characteristic. Frequently, 12 liters of normal
saline must be administered.
Acute pericarditis most frequently develops 24 days after the infarction. Free wall rupture, septal rupture,
and papillary muscle rupture typically do not occur until 15 days after the acute MI. Septal rupture is
more common with anterior MI.
Ref:

Tintinalli JE, Kelen GD, Stapczynski JS (eds): Emergency Medicine: A Comprehensive Study Guide, ed 7. McGraw-Hill,
2011, pp 383-384.

80

Item 240
ANSWER:

Eczema craquel, a common complication of aging, is due to dryness of the skin. It is best treated with
wet compresses and antibiotics to remove crusts and suppress infection, followed by topical corticosteroids
and lubricants. The primary lesions do not suggest scabies or necrobiosis lipoidica, and neurodermatitis
and lichen sclerosis are secondary responses to itching.
Ref:

Habif TP: Clinical Dermatology: A Color Guide to Diagnosis and Therapy, ed 5. Mosby Elsevier, 2010, p 110.

81

Вам также может понравиться